Pediatrics Examination Review
Pediatrics Examination Review
Pediatrics Examination Review
EXAMINATION REVIEW
Questions , Answers
And Teaching nots for
each question
Dr. kamel hassan
C.E.S DE PEDIATRIE, C.P.B. OF PEDIATRICS
Medical Services Directorate
PEDIATRICS
EXAMINATION REVIEW
PEDIATRICS
EXAMINATION REVIEW
INTRODUCTION
This is a test book, not a text book.
This test book will be a learning experience because the
questions and answers are structured to give you
experience in preparation for any examination.
In addition ,the questions and answers are structured to
teach a great deal about problems, disease, differential
diagnosis, process, and therapy.
It will test your medical skills and approach to diagnosis,
laboratory confirmation, therapeutic planning, and
anticipated outcome.
Pediatrics has been substantially revised and updated to
keep in step with current trends in medical education and
the continuing expansion of scientific knowledge.
The questions themselves are organized in broad
categories to give you a representative sampling of the
material covered in course work and are modeled as
closely as possible after those used by the board.
The answers and teaching notes follow each questions.
Using this test book, you may identify the areas of
strength and weakness in your command of the subject.
PEDIATRICS
EXAMINATION REVIEW
Questions
(PART 1)
TRUE OR FALSE
PEDIATRICS
EXAMINATION REVIEW
PEDIATRICS
EXAMINATION REVIEW
8- ACE inhibitors:
1. They cause hypotension via a drop in systemic vascular resistance
2. May lead to a side effect of dry cough, due to an increase in bradykinin levels
3. In the normal kidney, will cause an increase in renal blood flow
4. They are highly protein bound drugs
5. Indications include congestive cardiac failure in a patient with renal artery stenosis
PEDIATRICS
EXAMINATION REVIEW
14- The following is true regarding changes in the fetal cardiovascular system after
birth:
1. There is normally immediate closure of the ductus arteriosus
2. Hypoxia-induced vasoconstriction is the mechanism of closure of the ductus arteriosus
3. The ligamentum teres is the remnant of the umbilical vein
4. Regression of right ventricular hypertrophy occurs postnatally
5. Inferior vena caval pressure falls after birth
15- X-linked hyper IgM syndrome :1. Causes defective B cell development
2. Is associated with normal levels of IgG
3. Can present with pneumocystis carinii infection
4. Is due to a defective CD40 ligand
5. Is due to a failure of Ig isotype switching
PEDIATRICS
EXAMINATION REVIEW
19- The following syndromes commonly have associated renal manifestations:1. Zellwegers syndrome
2. Drash syndrome
3. Meckel Gruber syndrome
4. Noonan's syndrome
5. Chromosome 11p deletion
20- Bile:
1. Is secreted by the bile canaliculi
2. Contains NaHCO3
3. Is released from the gall bladder in response to the hormone gastrin
4. Contains salts that are only water soluble
5. Salts are mostly reabsorbed in the large bowel
21- The following are true regarding vaccine administration in children:1. Live vaccines are contraindicted within 3 weeks of another live vaccine administration
2. MMR can be given a week after administration of IV immunoglobulins
3. Live vaccines are not given to children on prolonged steroid therapy
4. Severe febrile illness is a contraindication for DPT
5. Severe malnutrition is a contraindication for BCG
24- T lymphocytes:1. Are derived from a precursor cell in the bone marrow
2. 95% die in the thymus
3. All have the alpha-beta heterodimeric T cell receptor
4. Are usually activated by intact protein antigens
5. Are prominent in type II hypersensitivity reactions
PEDIATRICS
EXAMINATION REVIEW
PEDIATRICS
EXAMINATION REVIEW
34-In paroxysmal nocturnal haemoglobinuria:1. The defect is a congenital abnormality of RBC production by the bone marrow
2. Hamoglobinuria is a characteristic finding
3. Venous thrombosis is a recognised complication
4. The diagnostic test is Hams test
5. Transforms into acute leukaemia in approx. 5%
35-Phosphodiesterase inhibitors:
1. Have an anti-diuretic effect
2. Cause bronchodilation
3. Cause peripheral dilatation
4. Cause relaxation of the diaphragm
5. Decreases the pulmonary capillary wedge pressure
36-Regarding mumps infection:1. The incubation period is 7-14 days
2. It is often asymptomatic
3. Children are not infectious after the development of parotitis
4. Infertility is a frequent complication of the orchitis associated with mumps
5. It is caused by a DNA virus
10
PEDIATRICS
EXAMINATION REVIEW
37-The following are known functions of T helper cells :1. Production of T cell growth factors such as IL-2
2. Promotion of immunoglobulin isotype class switching
3. Production of perforin
4. Production of suppressive cytokines such as IL-10, TGF-beta
5. Induction of the maturation of immature dendritic cells
38-Regarding immunisation in the UK:1. The first dose of vaccines is given at a corrected age of 2 months in case of premature delivery
2. Family history of febrile convulsion following immunisation is a relative contraindication
3. Measles, mumps, rubella and polio are contraindicated in HIV +ve children
4. Live vaccines should be avoided within 6 months of termination of lymphoma
treatment
5. Prolonged inconsolable crying to previous immunisation is a definite contraindication.
39-Regarding paediatric liver transplantation:1. The most common indication is failed Kasai procedure
2. 5-yr survival rates for children after orthotopic liver transplantation are around 30%
3. FK506 has superceded tacrolimus as an immunosuppressant
4. Signs of chronic liver disease persist despite improvement in liver function
5. Organ rejection is an early post operative complication
40-The following statements are correct regarding Horners syndrome:
11
PEDIATRICS
EXAMINATION REVIEW
43-Chronic lymphocytic leukaemia :1. Is associated with smear or smudge cells in the peripheral blood
2. Is asymptomatic at diagnosis in less than 10% of patients
3. Should be treated aggressively in the early stages of the disease
4. Has a median survival of less than 5 years
5. Is associated with splenomegaly at diagnosis in about 50% of patients
46-In asthma:1. The eosinophils to the lung are recruited from the bone marrow
2. Steroids are contra-indicated if the patient is a diabetic
3. Blocking interleukin-5 maybe therapeutic
4. Leukotriene inhibitors are associated with the development of Churg-Strauss .
5. Bronchial hyper-responsiveness (BHR) does not have to be present.
48-Heat inputs and outputs: Which of the following statements are true?
1. Changes in muscle activity are the main source of heat production for temperature control.
2. The skin is a good thermal insulator whose effectiveness is controlled through blood flow.
3. The blood flow through skin is controlled through vasodilator sympathetic nerves whose firing
rate is increased in response to heat.
4. Sweating can increase body evaporative water loss from 0.6 to over 4.5 litres per day and
eliminate 2.4 Mcal heat from the body.
5. Local humidity and air movement do not significantly influence heat loss.
12
PEDIATRICS
EXAMINATION REVIEW
50-Sarah is eight years old and she was referred by her G.P with a velvety,
pigmented rash on her neck and axilla. It is not itchy and she is not disturbed by it.
She is the only child of her parents. Her weight is 49 kg and her G.P was concerned
about this and has addressed this issue with her and her parents.
Which one of the following investigations is the most relevant for her future
management?
1. Fasting serum cholesterol/triglyceride
2. Fasting serum insulin/glucose ratio
3. Serum adrenocorticotrophic hormone (ACTH)/cortisol
4. Plasma leptin
5. Serum thyroid-stimulating hormone (TSH)
51-Congenital CMV infection:1. Only 10% of affected pregnancies have resulting long term sequele at birth
2. Diagnosis is by viral isolation from the urine
3. Hearing loss can deveolp gradually over the first 5 years
4. The affected newborn should be treated with ganciclovir
5. Intracranial calcifications are seen in a periventricular distribution
53-In scleroderma (systemic sclerosis):1. Skin tightness may improve with time
2. Females are affected three times more commonly than men
3. Calcinosis only occurs in CREST form of the disease
4. Anti-Scl 70 antibodies occur in 50-60% of cases
5. Penicillamine may delay the development of internal organ complications
13
PEDIATRICS
EXAMINATION REVIEW
55-Regarding schistosomiasis:
1. Freshwater snails are the definitive host
2. Cercariae penetrate the skin and may lead to an itchy papular rash
3. Fever, urticaria and eosinophilia may be seen in the initial illness due to antigen-antibody
complex formation
4. Schistosoma haematobium infection may lead to rectal bleeding
5. Schistosoma haematobium infection may lead to an obstructive uropathy
56-Regarding platelets:
1. They have a life span of approximately 10 days
2. Platelet production is controlled by specific cytokines
3. Platelets should be stored between 2 and 6 degrees C
4. Platelets can be stored for a maximum of 48 hours
5. A pool of platelets transfused can produce an increment of 50 000 / ml if platelet consumption
is not an issue
58-Carbon dioxide:
1. Does not affect plasma HCO3 concentration
2. Is transported in the blood as phosphate esters
3. Is transported in the blood in simple solution
4. Arterial CO2 rises during sleep
5. Arterial CO2 rises when living at an altitude of 9000 meters
14
PEDIATRICS
EXAMINATION REVIEW
61-The following statements are correct regarding urinary analysis in acute renal
failure:
1. There is a good correlation between functional renal impairment and the findings on urinalysis.
2. Pigmented 'muddy brown' granular casts suggest acute tubular necrosis.
3. Transparent hyaline casts are nonspecific and may be present in any case of ARF unrelated to
the specific aetiology.
4. RBC casts suggest primarily renal arteriolar diseases such as vasculitis.
5. Presence of proteinuria argues against acute tubular necrosis and suggests a primary
gomerular pathology.
64-The stiff-man syndrome :1. Is a syndrome associated with chronic inflammation of peripheral nerves.
2. Is a predominantly autoimmune syndrome.
3. Is uniquely associated with GAD antibodies.
4. Is associated with fibrillation and myokimeia.
5. Is best treated by benzodiazepenes and baclofen.
65-The following conditions will present with cyanosis in the first week of life:
1. Aortic stenosis
2. Transposition of the great vessels
3. Hypoplastic left heart syndrome
4. Fallot's tetralogy
5. Fallot's pentalogy.
66-Sturge-Weber syndrome :1. Occurs in 50% of children with port wine stains in the ophthalmic division of the trigeminal
nerve
2. Develop hemiplegia on the ipsilateral side of the vascular lesion
3. Are associated with cataracts
4. Typically have slit like ventricles on CT
15
PEDIATRICS
EXAMINATION REVIEW
69-A 9 yr old girl is seen in clinic. Her teacher at school has reported that she has
numerous episodes throughout the day of abruptly staring into space midconversation for a few seconds with rapid return of awareness. Which of the
following features suggest complex partial rather than absence seizures as the
cause of her problem :1. Reporting unusual smells prior to the episodes
2. Provocation of the episodes by hyperventilation
3. Abnormal features on MRI scanning of the brain
4. A good response to anti-epileptic drugs
5. A normal EEG
71-Regarding cytokines:1. IL-1 causes pyrexia and induces protein synthesis in the liver
2. Interferon gamma causes increased expression of MHC class II molecules on antigen
presenting cells
3. Chemokines can induce the transmigration of T cells into inflamed tissues
4. Can be used clinically to help treat infection
5. IL-2 s the main cytokine produced by nave T cells
16
PEDIATRICS
EXAMINATION REVIEW
2. Intravenous fluid
3. Throat swab
4. Intravenous antibiotic
5. Lumbar puncture
73-The following congenital lesions are associated with cyanosis:1. Aortopulmonary window
2. Total anomalous pulmonary venous drainage (TAPVD)
3. Coarctation of the aorta
4. Tetralogy of Fallot
5. Eisenmenger's syndrome
77-The following tumour markers are strongly associated with the tumours shown:
1. Alpha-FP and hepatocellular carcinoma.
2. Beta-HCG and choriocarcinoma.
3. CA-19.9 and ovarian carcinoma.
4. CA-125 and pancreatic carcinoma.
5. CEA and colorectal carcinoma.
17
PEDIATRICS
EXAMINATION REVIEW
2. Atrial fibrillation
3. Factor V Leiden mutation
4. Protein C deficiency
5. Polycythaemia
79-Salbutamol:
1. Can cause an increase in PEFR
2. Has no effect on the FEV1
3. Has no effect on hypoxic-pulmonary vasoconstriction
4. May cause a rise in insulin release
5. Causes hypokalemia after prolonged use
18
PEDIATRICS
EXAMINATION REVIEW
1. Marfan's syndrome
2. Rickets
3. Upper airway obstruction
4. Exercise limitation
5. Rightward shift of the heart
85-Renal ultrasound:
1. Is a reliable means of looking for vesico ureteric reflux (VUR).
2. Has a false negative result of 10 - 20% when looking for renal scarring.
3. Has a false negative result of 15 - 45% when looking for renal scarring.
4. Has a false positive rate of 25 - 50% when looking for VUR.
5. Is a good means of detecting distal ureteric dilation.
87-The following are true about acne:1. Females with acne commonly have elevated blood androgens
2. Oral steroids are a recognised cause
3. Interleukin 1 alpha promotes comedogenesis
4. Oral isotretinoin causes hyperlipidaemia
5. The enzyme Type 1 5 alpha reductase is expressed in sebaceous gland ducts
88-The following criteria must exist before a diagnosis of brain death can be made:
1. Fixed, dilated pupils unresponsive to light.
2. Absent gag reflex with pharyngeal stimulation.
3. Neuromuscular drugs should have been stopped for at least twelve hours.
4. No eye movement when 20 mls cold water is perfused into the external auditory meatus.
5. No motor response from painful stimuli.
19
PEDIATRICS
EXAMINATION REVIEW
92-Regarding ketamine:
1. Is contra-indicated in reversible airways obstruction
2. Is more emetogenic than thiopentone
3. Has anti-sialogogue properties
4. May be used safely in patients with suspected raised intra-cranial pressure
5. Is a potent respiratory depressant
20
PEDIATRICS
EXAMINATION REVIEW
100-Pulmonary surfactant
1. Is partly recycled by endocytosis into the synthesizing cell
2. Is produced by type 1 alveolar cells
3. Reduction in pulmonary flow can cause a decrease in surfactant production
4. Synthesis is inhibited by thyroxine
5. Synthesis is stimulated by glucocorticoids
21
PEDIATRICS
EXAMINATION REVIEW
104-The following are true of COPD exacerbations:1. All patients should be given antibiotics
2. Are associated both with rhinovirus and chlamydia pneumoniae infection
3. There is a positive correlation in mortality and the number of smoking pack years
4. pH less than 7.35 is an indication for antibiotics according to the BTS guidelines
5. Moraxhella catarrhalis is the most frequently implicated organism in exacerbation
22
PEDIATRICS
EXAMINATION REVIEW
108-A 16 year old girl is admitted with progressive weight loss for which no
organic cause can be found. The following factors would be against a diagnosis of
anorexia nervosa:1. A history of laxative abuse
2. The delusion that someone is trying to control her eating patterns
3. An excess of physical exercise
4. Self-induced vomiting
5. Low plasma potassium
110-The following poisons are matched to the appropriate therapy:1. Carbon monoxide - hyperbaric oxygen
2. Organophosphates - atropine and pralidoxime
3. Beta-blocker - phentolamine
4. Methanol - ethanol
5. Tricyclic antidepressants - phenytoin
112-Restriction fragment length polymorphisms (RFLPs):1. Are found every 100 bases
2. Are useful in diagnosis of multifactorial diseases
3. Are useful in linked-gene diseases
4. Are generated by digestion of DNA with bacterial enzymes
5. Are detected by Northern blotting
23
PEDIATRICS
EXAMINATION REVIEW
118-The following statements are true for the Renin Angiotensin System:1. Renin converts Angiotensin I to Angiotensin II in the kidney
2. Angiotensin II promotes renal arteriolar dilatation
3. Angiotensin II inhibits aldosterone secretion by the adrenal glands
4. Angiotensin II receptors are blocked by spironolactone
5. Losartan inhibits formation of Angiotensin II
24
PEDIATRICS
EXAMINATION REVIEW
4. Ataxia
5. Seizure
122-Regarding Parvovirus infections in children:1. It is the causative agent for exanthem subitum
2. Arthritis is a very common manifestation in children
3. Chronic anemia can be a manifestation in some children
4. Infection in pregnancy can result in fetal anemia
5. Aplastic crises due to parvovirus infection in sickle cell disease usually requires bone marrow
transplant
124-Regarding Gross Motor Development, a normal child would be expected to:1. Roll over by 2 months
2. Ride a tricycle at 2 years
3. Kick a ball at 2 years
4. Cruise by 1 year
5. Not run until 2 years
25
PEDIATRICS
EXAMINATION REVIEW
127-Urine
1. normal pH is >5.5
2. normal Na content is <20mmol/L
3. normal osmolality is 300-600 mosmol/kg
4. protein content of 500mg/day is within normal range
5. functional minimum urine output/day in 70kg male is 1400mls
129-Lung surfactant
1. Decreases the surface tension within an alveolus
2. Causes an increase in chest wall compliance
3. Is a glycoprotein
4. Maintains the same surface tension for different sized alveoli
5. Appears only after the 1st week of life
130-With regards to bronchiolitis, the following are true :1. It is linked to maternal smoking.
2. RSV immune globulin is recommended in infants with cyanotic congenital heart disease to
prevent severe RSV infection.
3. Ribavarin is curative.
4. Lymphopenia is common.
5. CXR commonly shows hyperinflation.
26
PEDIATRICS
EXAMINATION REVIEW
137-Regarding lead poisoning:1. Urinary lead concentration is the most useful indicator of exposure
27
PEDIATRICS
EXAMINATION REVIEW
142-A patient develops low back pain which is progressive. He is found to a spinal
schwannoma. It is resected, but division of the posterior spinal root occurs.
Which of the following is likely to result from this?
1. Loss of motor function
2. Loss of tendon reflex
3. Loss of pain, temperature and preservation of light touch
4. Spinothalamic tract degeneration above the section level
5. Absence of sweating in the limb
28
PEDIATRICS
EXAMINATION REVIEW
145-Chronic lymphocytic leukaemia :1. Is associated with smear or smudge cells in the peripheral blood
2. Is asymptomatic at diagnosis in less than 10% of patients
3. Should be treated aggressively in the early stages of the disease
4. Has a median survival of less than 5 years
5. Is associated with splenomegaly at diagnosis in about 50% of patients
146-In asthma:1. The eosinophils to the lung are recruited from the bone marrow
2. Steroids are contra-indicated if the patient is a diabetic
3. Blocking interleukin-5 maybe therapeutic
4. Leukotriene inhibitors are associated with the development of Churg-Strauss .
5. Bronchial hyper-responsiveness (BHR) does not have to be present.
147-With regards severe combined immunodeficiency:1. Patients have defective T-cell dependent antibody responses and defective cell-mediated
immunity
2. Patients are susceptible to infection with bacteria and viruses
3. X-linked SCID is caused by a defect in the IL-2 receptor
4. ADA (adenosine deaminase) and PNP (purine nucleotide phosphorylase) deficiency cause
accumulation of nucleotide metabolites that are toxic to developing T-cells resulting in T-cell and
B-cell deficiencies and a SCID phenotype
5. SCID can be due to an autosomally inherited defect in DNA repair analogous to the genetic
defect in the SCID mouse strain
29
PEDIATRICS
EXAMINATION REVIEW
150-The following statements concerning congenital heart disease (CHD) are true:1. VSDs account for 70% of all CHD.
2. ASDs account for 40% of all CHD
3. Approx. 13% of neonates with chromosomal abnormalities have associated CHD
4. Approx. 80% of children with Downs Syndrome have CHD
5. Approx. 40% of patients with Turners Syndrome have coarctation and-or aortic stenosis.
151-The following are clues to the cause of an upper gastrointestinal (GI) bleed:
1. Purpuric rash suggestive of Henoch-Schonlein purpura (HSP)
2. Hypopigmented spots on the lips suggestive of Peutz-Jeghers syndrome
3. Webbed neck and low hairline
4. Perianal skin tags
5. Haematochezia due to peptic ulceration
30
PEDIATRICS
EXAMINATION REVIEW
31
PEDIATRICS
EXAMINATION REVIEW
161-Regarding complex partial epilepsy:1. Absence seizures and generalised tonic-clonic seizures may occur
2. Consciousness is altered
3. Hyperventilation often provokes a complex partial seizure
4. EEG changes disappear in sleep
5. Ethosuximide is an affective treatment
164-Neostigmine:
1. Has been used in the treatment of SVT
2. Prolongs the duration of action of suxamethonium
3. Has an oral bioavailability of 50%
4. Is metabolized by plasma esterases
5. Crosses the placenta
32
PEDIATRICS
EXAMINATION REVIEW
166-The following statements are true:1. A moro reflex with the fists remaining clenched is normal.
2. The plantar grasp persists longer than the palmar grasp
3. Deep tendon reflexes are difficult to elicit at birth
4. An extensor Babinski response is normal in the first year
5. A newborn baby from 32 weeks gestation will turn towards a light
167-Glucocorticoids:
1. Are secreted from the zona fasciculata within the adrenal cortex.
2. Molecules consist of 21 carbon atoms.
3. Cause retention of sodium.
4. Inhibit gluconeogenesis.
5. Reduce lymphocyte counts.
168-In the treatment of attention deficit hyperactivity disorder :1. Behavioural therapy is never useful
2. Pharmacological intervention includes the use of stimulant drugs such as amphetamines
3. Methylphenidate (Ritalin) should not be given later than 4pm
4. Parenting techniques can be taught
5. Poor appetite is a common side affect of methylphenidate (Ritalin)
170-The following diseases are inherited in an autosomal recessive manner:1. Insulin dependant diabetes
2. Primary ciliary dyskinesia
3. Batten's disease
4. Congenital adrenal hyperplasia
5. Pyloric stenosis
171-Noradrenaline
1. Is a neurotransmitter at the pre-ganglionic sympathetic nervous system
2. Is produced by the adrenal cortex
3. May cause a reflex bradycardia when given by infusion
4. Causes a rise in metabolic rate
5. It acts predominantly via the beta adrenoceptor
33
PEDIATRICS
EXAMINATION REVIEW
172-In a healthy baby the transition from fetal to neonatal circulation involves:1. Functional closure of the foramen ovale in the first 24 hours
2. Blood flow in the ductus arteriosus continues from right to left until its closure
3. Decrease in pulmonary artery resistance following closure of the ductus arteriosus
4. The ductus arteriosus closes in response to decreased oxygen concentrations
5. The umbilical artery is a branch of the common iliac artery
173-The pancreas:
1. Is entirely retroperitoneal
2. Is supplied by branches of the coeliac artery alone
3. Is crossed by the attachment of the transverse mesocolon
4. Is crossed by the left renal vein
5. Is related to the common bile duct posteriorly
175-Abnormalities associated with foetal alcohol syndrome include:1. Short palpebral fissures
2. Maxillary hyperplasia
3. Hyperactivity in childhood
4. Intra uterine growth retardation
5. Hypotonia
34
PEDIATRICS
EXAMINATION REVIEW
35
PEDIATRICS
EXAMINATION REVIEW
185-Conn's syndrome:
1. Is more common in females.
2. Is due to atrophy of zona glomerulosa.
3. Causes muscle weakness.
4. Leads to sodium retention.
5. Leads to increased levels of plasma angiotensinogen 2.
187-Phagocytic Cells :1. Have receptors that recognise bacterial cell wall constituents
2. Require clonal expansion to mount an effective response
3. Release cytokines wich affect the adaptive immune system
4. Are important for the resolution of inflammation
5. Initiate firm attachment by binding to ICAM on the endothelium
36
PEDIATRICS
EXAMINATION REVIEW
37
PEDIATRICS
EXAMINATION REVIEW
3. Haemolytic disease.
4. Cytomegalovirus infection.
5. Cystic fibrosis.
199-The following maternal conditions are known to cause adverse effects on the
neonate:
1. Diabetes insipidus.
2. Toxoplasmosis.
3. Chronic myeloid leukaemia.
4. SLE.
5. Hyperthyroidism.
38
PEDIATRICS
EXAMINATION REVIEW
39
PEDIATRICS
EXAMINATION REVIEW
40
PEDIATRICS
EXAMINATION REVIEW
41
PEDIATRICS
EXAMINATION REVIEW
drugs.
1. A compensated metabolic acidosis is achieved via hyperventilation, which will decrease the PaCO2.
As it is acute there will be no change in the CSF pH.
2. Studies have shown that although there is the initial cerebral vasoconstriction with the low PaCO2,
this gradually normalizes within 72 hours despite the low PaCO2.
4. The CBF shows an exponential increase in the CBF at PaO2 levels of below 5.5 Kpa but is fairly
constant at levels above that.
5. Every decrease in body temperature of 1 degree celcius will decrease the cerebral metabolism by
7%.
42
PEDIATRICS
EXAMINATION REVIEW
8- ACE inhibitors:
1. They cause hypotension via a drop in systemic vascular resistance
2. May lead to a side effect of dry cough, due to an increase in bradykinin levels
3. In the normal kidney, will cause an increase in renal blood flow
4. They are highly protein bound drugs
5. Indications include congestive cardiac failure in a patient with renal artery stenosis
Teaching Notes for Question 8
Theme: ACE inhibitors
* ACE inhibitors cause a fall in afterload or SVR, which leads to hypotension (especially on
the first dose) but may increase cardiac output in those with heart failure.
* They also prevent the breakdown of bradykinin, which leads to dry cough in many
patients.
* They are generally poorly protein bound and readily removed by hemodialysis.
* Although in the normal kidney, they will increase renal blood flow and thus GFR, in a
patient with renal artery stenosis, they reduce GFR and can precipitate acute renal failure
and flash pulmonary edema; thus they are contraindicated in patients with renal artery
stenosis.
43
PEDIATRICS
EXAMINATION REVIEW
44
PEDIATRICS
EXAMINATION REVIEW
3. Reidel's thyroiditis.
4. Myasthenia gravis.
5. Motor neurone disease.
Teaching Notes for Question 11
Theme: Auto antibodies
Reidel's thyroiditis is probably viral in origin.
In myasthenia, there are circulating antibodies against acetylcholine receptors at the
neuromuscular junction.
The aetiology of motor neurone disease is not clear.
12-The following can cause cardiomyopathy in HIV disease:1. Mycobacterium tuberculosis
2. Lymphoma
3. Kaposi Sarcoma
4. CMV infection
5. HIV of the heart
Teaching Notes for Question 12
Theme: Cardiomyopathy in HIV
Cardiomyopathy can be caused by HIV itself, or other infections such as CMV, toxoplasma.
However, although TB, Kaposi sarcoma, and lymphoma can cause an apparent
cardiomegaly on a chest radiograph, this is due to pericardial effusions rather than
cardiomyopathy. A pericardial effusion carries a poor prognosis in HIV. HIV cardiomyopathy
is linked to CD4 count and HIV encephalopathy.
13- Gastric emptying is slowed by:
1. Supine posture
2. Gastric distention
3. Domperidone
4. Ranitidine
5. Cisapride
Teaching Notes for Question 13
Theme: Gastric emptying
2,3,5 - Increases emptying. 4 - No effect
14- The following is true regarding changes in the fetal cardiovascular system after birth:
1. There is normally immediate closure of the ductus arteriosus
2. Hypoxia-induced vasoconstriction is the mechanism of closure of the ductus arteriosus
3. The ligamentum teres is the remnant of the umbilical vein
4. Regression of right ventricular hypertrophy occurs postnatally
5. Inferior vena caval pressure falls after birth
45
PEDIATRICS
EXAMINATION REVIEW
46
PEDIATRICS
EXAMINATION REVIEW
47
PEDIATRICS
EXAMINATION REVIEW
48
PEDIATRICS
EXAMINATION REVIEW
49
PEDIATRICS
EXAMINATION REVIEW
.Peritoneal or hemodialysis .6
Note- Calcium gluconate stabilizes the myocardium by antagonising the effect of potassium
.on the heart. It does not lower serum K+ levels
24- T lymphocytes:1. Are derived from a precursor cell in the bone marrow
2. 95% die in the thymus
3. All have the alpha-beta heterodimeric T cell receptor
4. Are usually activated by intact protein antigens
5. Are prominent in type II hypersensitivity reactions
Teaching Notes for Question 24
Theme: T lymphocytes
T lymphocytes comprise 70-80% of the lymphocyte population and are haematopoietic
cells. They are produced in bone marrow and paracortical regions of lymph nodes and
spleen. Maturation occurs in the thymus, and involves a complicated series of positive and
negative selection steps coordinated by the thymocytes and dendritic cells within the
thymus, resulting in apoptosis of unwanted cells. The TCR is a heterodimer of two chains
(alpha and beta, or gamma and delta) linked by disulphide bonds and is found at the cell
surface associated with a complex of other polypeptides known collectively as CD3. The
two types of TCR are called TCR-1 (gamma-delta) and TCR-2 (alpha-beta). TCR-1 cells are
thought to have a restricted repertoire and to be mainly non-MHC restricted. TCR-2 cells
.recognise peptide fragments associated with MHC molecules
50
PEDIATRICS
EXAMINATION REVIEW
51
PEDIATRICS
EXAMINATION REVIEW
cardiomyopathy:1. The prognosis is related to the degree of left ventricular outflow gradient
2. Prognosis is worse in children
3. Runs of Non sustained ventricular tachycardia on Holter have a high predictive value
for sudden cardiac death
4. Progresses to dilated cardiomyopathy in 10% cases
5. Strenuous activity is proscribed in all patients
Teaching Notes for Question 28
Theme: Hypertrophic obstructive cardiomyopathy
In hypertrophic cardiomyopathy, the degree of outflow obstruction and the risk of
sudden death depend on the gene defect. There is no correlation betreen these
two features however. Presentation in childhood signifies worse prognosis. Runs of
non-sustained VT are found in one-fourth of patients of HCM patients, although
sustained VT is uncommon. In some, it is a harbinger of sudden cardiac death;
however, its overall predictive value in identifying patients at high risk of sudden
death is quite limited. Treatment with beta-blockers and amiodarone may be of
.benefit as may implantation of a defibrillator
29- Disseminated intravascular coagulation:
1. Is usually associated with a high fibrinogen level
2. Is usually associated with a normal thrombin time
3. May be associated with cytomegalovirus or varicella zoster infections
4. May occur in liver disease
5. May be associated with formation of microthrombi, so should be treated with
intravenous heparin
Teaching Notes for Question 29
Theme: Disseminated intravascular coagulation
* Disseminated intravascular coagulation (DIC) may be caused by bacterial infections, in
particular gram-negative and meningococcal septicemia, by severe falciparum malaria
infections and by viral infections including CMV, HIV and varicella zoster.
* Liver disease can also cause DIC due to release of thromboplastins from damaged liver
cells
and
reduced
concentrations
of
antithrombin
and
protein
C.
* Investigations show a prolonged thrombin time, prothrombin time and activated partial
thromboplastin time (aptt), thrombocytopenia, a low fibrinogen and raised D-dimers or
FDPs.
* DIC may be associated with microthrombin causing skin lesions, renal failure, gangrene
or cerebral ischemia but heparin therapy is usually not indicated as it can aggravate the
bleeding tendency.
30-Cardiac tamponade:
1. Occurs only after penetrating injuries to the heart.
2. Is characterised by arterial hypertension.
3. The central venous pressure is raised.
4. Should be treated with aspiration using a needle inserted via the second intercostal
space, mid-clavicular line.
52
PEDIATRICS
EXAMINATION REVIEW
53
PEDIATRICS
EXAMINATION REVIEW
Somatic gene therapy (i.e. addition of a gene to the somatic cells or tissues rather than
the germ line) is ideally suited for monogenic disorders caused by loss of gene function
where even partial restoration of the protein will greatly ameliorate the symptoms, as this
form of gene therapy is unlikely to restore the levels of the protein to normal. Adenosine
deaminase deficiency, haemoglobinopathies in particular thalassaemia major, and
haemophilia fit these criteria. Patients with diabetes mellitus require controlled release of
insulin and gene therapy is unlikely to provide the answer. Islet cell transplantation into the
portal vein is more promising as it preserves the microenvironment of the beta cells of the
islet and allows controlled temporal release of insulin.
35-Phosphodiesterase inhibitors:
1. Have an anti-diuretic effect
2. Cause bronchodilation
3. Cause peripheral dilatation
4. Cause relaxation of the diaphragm
5. Decreases the pulmonary capillary wedge pressure
54
PEDIATRICS
EXAMINATION REVIEW
38-Regarding immunisation in the UK:1. The first dose of vaccines is given at a corrected age of 2 months in case of
premature delivery
2. Family history of febrile convulsion following immunisation is a relative
contraindication
55
PEDIATRICS
EXAMINATION REVIEW
3. Measles, mumps, rubella and polio are contraindicated in HIV +ve children
4. Live vaccines should be avoided within 6 months of termination of lymphoma
treatment
5. Prolonged inconsolable crying to previous immunisation is a definite contraindication.
Teaching Notes for Question 38
Theme: Immunisations in childhood
Prematurity, asthma, chronic lung disease, congenital heart disease, and Downs
syndrome are associated with an increased risk of complications from infectious
diseases and they should be immunised as a matter of priority. Premature babies
should be immunised according to the recommended schedule from 2 months
after birth, irrespective of prematurely. Studies have shown that antibody
response and adverse events are not significantly different in preterm and term
infants immunised 2,3, and 4 months afterbirth. Prolonged inconsolable crying or
high pitched screaming more than 4 hours is classified as a severe reaction to
immunisation and is a contraindication to subsequent immunisation with same
vaccine. Patients on active chemo or radiotherapy or within 6 months of
termination of such treatment are contraindicated for live vaccines. They may not
be able to produce a normal immune response to live vaccines and could
potentially suffer from severe manifestation such as disseminated infection with
BCG or paralytic polio from vaccine virus. No harmful effects have been reported
with MMR and polio in HIV +ve individuals. However it should be noted that Polio
is secreted for longer periods in HIV positive patients as compared to other
people. Family history of febrile convulsion is not a contraindication to
immunisation.
39-Regarding paediatric liver transplantation:1. The most common indication is failed Kasai procedure
2. 5-yr survival rates for children after orthotopic liver transplantation are around 30%
3. FK506 has superceded tacrolimus as an immunosuppressant
4. Signs of chronic liver disease persist despite improvement in liver function
5. Organ rejection is an early post operative complication
Teaching Notes for Question 39
Theme: Liver transplantation
Paediatic liver transplantation is an uncommon procedure (about 500/year in US)
but the most common reason is following a failed portoenterostomy (Kasai)
procedure following extrahepatic biliary atresia. The next most common
indications are metabolic liver disease and acute hepatic necrosis. Orthotopic
(same species) transplants are surprisingly successful especially with improved
immunosuppression and better preservation of the organ (up to 18 hr ex vivo
with <2% primary nonfunction). Most recent figures are 81% at 1 year and 77%
at 5 years. As liver function improves, so does the childs growth stigmas of
chronic liver disease resolve. Organ rejection tends to occur after the early phase
along with infection. Early complications include fluid shifts, electrolyte imbalance,
renal dysfunction, and hypertension. Vascular complications, such as thrombosis
of graft vessels, also may be an ominous early problem. Tacrolimus and FK506
are the same drug. Steroids and either cyclosporine or tacrolimus are standard
therapy to prevent rejection. Compared with cyclosporin, tacrolimus is associated
with lower rates of acute and chronic rejection and reduced use of corticosteroids.
40-The following statements are correct regarding Horners syndrome:
56
PEDIATRICS
EXAMINATION REVIEW
41-In HIE or Jobs syndrome :1. There is a markedly raised IgE level
2. There is an association with coarse facies, kyphoscoliosis, osteoporosis, and eczema
3. There is a low incidence of atopy due to defective IgE function
4. The condition is X-linked
5. Cold abscesses occur
Teaching Notes for Question 41
Theme: HIE or Jobs syndrome
HIE (HyperIgE syndrome) or Jobs syndrome is an autosomal dominant condition
characterised by coarse facies, kyphoscoliosis, osteoporosis, atopy and eczema.
Patients develop recurrent sinopulmonary and cutaneous infections that tend to
be much less inflamed than appropriate for the degree of infection (cold
abscesses). The IgE level is markedly raised in these patients
42-The following increases the pulmonary vascular resistance
1. Maximal inspiration
2. Maximal expiration
3. Anemia
4. Salbutamol
5. Prostacyclin
Teaching Notes for Question 42
Theme: Pulmonary circulation
The PVR is increased by
* Hypoxia, hypercarbia, acidosis.
* 5HT, histamine, vasoconstrictors.
* Alpha-adrenergic drugs.
The PVR is decreased by
57
PEDIATRICS
EXAMINATION REVIEW
* Vasodilators, Ach.
* Beta-adrenergic drugs.
* Parasym nervous sys
* Prostacyclin
Effect of respiration:
PVR varies with lung volume in a U-shaped curve. The lowest PVR is at the FRC
43-Chronic lymphocytic leukaemia :1. Is associated with smear or smudge cells in the peripheral blood
2. Is asymptomatic at diagnosis in less than 10% of patients
3. Should be treated aggressively in the early stages of the disease
4. Has a median survival of less than 5 years
5. Is associated with splenomegaly at diagnosis in about 50% of patients
Teaching Notes for Question 43
Theme: Chronic lymphatic leukaemia
Chronic lymphatic leukaemia (CLL) is due to an accumulation of mature lymphocytes in the
peripheral blood, bone marrow, spleen, liver and lymph nodes. Smear or smudge cells are
often seen in the peripheral blood which represent the fragile malignant cells which have
been squashed as the blood film is spread. It is asymptomatic at diagnosis in over 30% of
patients, and splenomegaly is seen in 50% at diagnosis. It is often a slowly progressive
disease and should not be treated in the early stages. Median survival is 3-5 years, but one
third of patients die of causes other than their CLL.
44-The following agents have good activity against Bacteroides fragilis
1. Benzylpenicillin
2. Piperacillin / tazobactam
3. Cefoxitin
4. Metronidazole
5. Chloramphenicol
Teaching Notes for Question 44
Theme: Anaerobic Infections
B fragilis produces a -lactamase which is sensitive to inhibitors. It also produces a
cephalosporinase, but it does not hydrolyse cefoxitin (cephalosporin with anaerobic
activity).
45-Digestion and absorption: Which of the following statements are true?
1. The glucose-containing polysaccharides starch, cellulose and glycogen are broken down for
absorption by salivary and pancreatic amylases.
2. Trypsin, chymotrypsin, carboxypeptidases and amino peptidases are responsible for the
breakdown of proteins for absorption.
3. Amphipathic phospholipids and bile salts convert micelles into lipid droplets ready for
absorption.
4. The three known fat soluble vitamins A, D and E are taken up by fat absorbing processes in
the gut.
5. About 9 litres of water enters the gut each day.
Teaching Notes for Question 45
Theme: Gastrointestinal system
58
PEDIATRICS
EXAMINATION REVIEW
59
PEDIATRICS
EXAMINATION REVIEW
60
PEDIATRICS
EXAMINATION REVIEW
consensus sequence in the ARE is the AUUUA repeat element. This sequence is found in
the 3' noncoding region of many eukaryotic mRNAs particularly those of transiently
expressed proteins such as cytokines and growth factors. The ARE triggers destabilization
of mRNA when bound by certain proteins. AREs have also been shown to play a role in the
stabilization of mRNA - MK2 (a protein kinase induced by LPS) may inhibit the binding or
activity of destabilising proteins associated with the AREs of proinflammatory cytokine
mRNAs. The complex mechanisms of ARE-mediated post-transcriptional regulation are still
being revealed.
50-Sarah is eight years old and she was referred by her G.P with a velvety, pigmented rash
on her neck and axilla. It is not itchy and she is not disturbed by it. She is the only child of
her parents. Her weight is 49 kg and her G.P was concerned about this and has addressed
this issue with her and her parents.
Which one of the following investigations is the most relevant for her future management?
1. Fasting serum cholesterol/triglyceride
2. Fasting serum insulin/glucose ratio
3. Serum adrenocorticotrophic hormone (ACTH)/cortisol
4. Plasma leptin
5. Serum thyroid-stimulating hormone (TSH)
Teaching Notes for Question 50
Theme: Diabete mellitus
Acanthosis nigrans is associated with insulin resistance or poor control and is indicated by
a serum insulin/glucose ratio.
51-Congenital CMV infection:1. Only 10% of affected pregnancies have resulting long term sequele at birth
2. Diagnosis is by viral isolation from the urine
3. Hearing loss can deveolp gradually over the first 5 years
4. The affected newborn should be treated with ganciclovir
5. Intracranial calcifications are seen in a periventricular distribution
61
PEDIATRICS
EXAMINATION REVIEW
62
PEDIATRICS
EXAMINATION REVIEW
warranted.
55-Regarding schistosomiasis:
1. Freshwater snails are the definitive host
2. Cercariae penetrate the skin and may lead to an itchy papular rash
3. Fever, urticaria and eosinophilia may be seen in the initial illness due to antigen-antibody
complex formation
4. Schistosoma haematobium infection may lead to rectal bleeding
5. Schistosoma haematobium infection may lead to an obstructive uropathy
Teaching Notes for Question 55
Theme: Schistosomiasis
Schistosomiasis is a trematode (fluke) infection.
Intermediate host - snails
Definitive host - humans
S. Haematobium
Urinary schistosomiasis
(Africa, Middle East)
Worms reside in vesical plexus
Eggs shed in urine
S. Mansoni
Intestinal/liver schistosomiasis
(Africa, SW Asia, Americas)
Worms reside in mesenteric veins
S. Japonicum
Eggs shed in stool
(China, Japan - more severe)
S. Intercalatum
(West/Central Africa)
S. Mekongi
(Mekong Delta)
LIFE CYCLE:
Eggs are shed in urine or stool into fresh water. Miracidia hatch and infect freshwater snails
(intermediate hosts). 4-8 weeks later cercariae emerge which penetrate skin of humans
and migrate via the lungs to the liver as a schistosomule. Adult worms develop in 1-3
months in the portal veins and migrate to their final habitats (see above) where eggs are
laid.
PATHOLOGY is caused by:
1. Invasion of cercariae:
* Swimmer's itch (papular dermatitis)
2. Katayama fever:
* Acute infection
* Fever, urticaria, eosinophilia, cough/wheeze, hepatosplenomegaly, diarrhoea
* Can treat with steroids
3. Eggs cause most of the pathology:
a) Eggs leaving gut/urinary tract cause bleeding
b) Eggs remaining cause granulomata and calcify:
* Urinary tract - obstructive uropathy, calcified bladder,
* squamous cell carcinoma bladder, pseudopapillomas
* GI tract - colonic polyposis leading to iron deficiency anaemia
c) Eggs in the portal tract cause periportal fibrosis (Symmers fibrosis) leading to portal
hypertension (but normal liver function)
d) Eggs can embolise to lungs (pulmonary hypertension, corpulmonale) or CNS (space
occupying lesion, cord compression)
4. Immune complex disease:
* Glomerulonephritis
* Nephrotic syndrome
DIAGNOSIS:
1. Direct microscopy of eggs in URINE (max at 12 noon, eggs have terminal spines remember terminal haematuria too) or STOOL (need to concentrate stools by formol-ether
method; eggs have lateral spines)
2. Immunodiagnostic tests - problematic: no indication of severity of infection; do not tell
past vs present infection; not species-specific; hi-tech.
TREATMENT:
Metriphonate - S. haematobium only; x3 doses
63
PEDIATRICS
EXAMINATION REVIEW
Praziquantel - all species, single dose (sometimes repeated), no serious toxicityct - colonic
polyposis leading to iro
56-Regarding platelets:
1. They have a life span of approximately 10 days
2. Platelet production is controlled by specific cytokines
3. Platelets should be stored between 2 and 6 degrees C
4. Platelets can be stored for a maximum of 48 hours
5. A pool of platelets transfused can produce an increment of 50 000 / ml if platelet
consumption is not an issue
Teaching Notes for Question 56
Theme: Platelets
Platelets are produced from megakaryocytes, which are regulated by thrombopoietin, a
specific cytokine. They have a lifespan of 10-12 days thereafter being destroyed in the
spleen.
Platelet concentrates should be stored at around 20 degrees C and the pH kept between
6.2 and 7.8 - these conditions reduce the risk of a change in morphology of the platelets.
They should also be continuously agitated to encourage gas exchange.
One pool of platelets gives on average, an increment of 10 000 / ml.
57-Concerning Salmonella infection:
1. Results in a non-invasive enteric infection
2. Infection invariably occurs following oral ingestion of contaminated food products
3. Statutory notification is required
4. Antibiotic treatment is required for Salmonellosis
5. With typhoid 15-20% become chronic carriers
Teaching Notes for Question 57
Theme: Salmonella infections
The risk of infection following ingestion of contaminated food or water is higher in the
immunosuppressed, elderly, sickle cell disease and those with reduced gastric acidity /
motility e.g. postgastrectomy, hypochlorhydria. Salmonellae are highly invasive but most
infections are mild and self limiting. Less than 5% of typhoid cases become chronic carriers
58-Carbon dioxide:
1. Does not affect plasma HCO3 concentration
2. Is transported in the blood as phosphate esters
3. Is transported in the blood in simple solution
4. Arterial CO2 rises during sleep
5. Arterial CO2 rises when living at an altitude of 9000 meters
Teaching Notes for Question 58
Theme: Carbon dioxide
1, 2, 3. CO2 is transported in blood in a variety of ways:
- Hydrolyzed to carbonic acid in the red cells, which then dissociates to hydrogen ions and
bicarbonate. Bicarbonate then diffuses into plasma in exchange for Cl ions.
- Dissolved in simple solution in plasma and RBC.
64
PEDIATRICS
EXAMINATION REVIEW
65
PEDIATRICS
EXAMINATION REVIEW
66
PEDIATRICS
EXAMINATION REVIEW
PEDIATRICS
EXAMINATION REVIEW
68
PEDIATRICS
EXAMINATION REVIEW
69-A 9 yr old girl is seen in clinic. Her teacher at school has reported that she has
numerous episodes throughout the day of abruptly staring into space mid-conversation for
a few seconds with rapid return of awareness. Which of the following features suggest
complex partial rather than absence seizures as the cause of her problem :1. Reporting unusual smells prior to the episodes
2. Provocation of the episodes by hyperventilation
3. Abnormal features on MRI scanning of the brain
4. A good response to anti-epileptic drugs
5. A normal EEG
Teaching Notes for Question 69
Theme: Childhood seizures
Complex partial seizures may present with brief periods of reduced awareness. Features
which make complex partial seizures, rather than absence seizures likely, include:
Later onset, longer duration, the presence of aura and post-ictal phenomena, focal
abnormalities or normal EEG (absences tend to have typical 3HZ spike pattern), associated
MRI abnormalities (mesial temporal sclerosis, for example) and a poor response to antiepileptic therapies.
70-The following statements are reagrding asthma:
1. Leukotriene antagonists are a first line treatment in asthma
2. The diagnosis of asthma involves blood tests
3. Treatment can be divided into those that prevent and those that relieve symptoms
4. Inhaled glucocorticoids improve lung function in asthmatics
5. Peak flow monitoring can be used in most children aged 5 years and above
Teaching Notes for Question 70
Theme: Asthma
Asthma remains a clinical diagnosis without a highly specific or sensitive test. The most
reliable investigations are those of lung function that show reversibility of bronchospasm.
Blood tests are not usually helpful. Peak flow monitoring is an easy way to monitor some
aspects of lung function and can be used in most children >5 years and in some who are
younger.
Treatments for asthma continue to evolve, but the two main areas remain those of
69
PEDIATRICS
EXAMINATION REVIEW
symptom relief and those that prevent attacks, bronchodilators and glucocorticoids
respectively. Glucocorticoids can improve lung function with regular use. Leukotriene
inhibitors have a useful role in some patients, but are not a first line treatment.
71-Regarding cytokines:1. IL-1 causes pyrexia and induces protein synthesis in the liver
2. Interferon gamma causes increased expression of MHC class II molecules on antigen
presenting cells
3. Chemokines can induce the transmigration of T cells into inflamed tissues
4. Can be used clinically to help treat infection
5. IL-2 s the main cytokine produced by nave T cells
Teaching Notes for Question 71
Theme: Cytokines
IL1 is known as endogenous pyrogen. It is produced mainly by activated mononuclear
phagocytes, but is also secreted by other cells e.g. epithelial cells and endothelial cells. At
low concentration, IL1 mediates local inflammation. At high concentrations, it causes
pyrexia and enhances acute phase protein synthesis. IFN-gamma is produces by activated
CD4+, CD8+ and NK cells. It is a potent activator of macrophages and promotes the
differentiation of Th1 cells. Chemokines are chemoattractant cytokines and recruit immune
and other haemopoietic cells to sites of inflammation
Cytokines are increasingly used in the therapeutic arena e.g. G-CSF is commonly used to
treat neutropaenic sepsis; IFN-alpha is used as an anti-viral agent, and IFN-beta is used to
treat multiple sclerosis. IFN-gamma is used to improve macrophage function in patients
with chronic granulomatous disease.
72-15-year-old teenager presented with few haehorrhagic spots, agitation and confusion
having been to school. Temp 39, pulse 110 and BP 120/80. The first management step
would be :1. Oxygen
2. Intravenous fluid
3. Throat swab
4. Intravenous antibiotic
5. Lumbar puncture
Teaching Notes for Question 72
Theme: Systemic Sepsis
The history suggets meningococcal sepsis, IV antibiotics must be given immedietly.
73-The following congenital lesions are associated with cyanosis:1. Aortopulmonary window
2. Total anomalous pulmonary venous drainage (TAPVD)
3. Coarctation of the aorta
4. Tetralogy of Fallot
5. Eisenmenger's syndrome
70
PEDIATRICS
EXAMINATION REVIEW
71
PEDIATRICS
EXAMINATION REVIEW
4. Myopathy
5. Decreased sensitivity to opioids
Teaching Notes for Question 76
Theme: Myxedema
* Hypothyroidism (myxedema) can be due either to pituitary disease or autoimmine
thyroiditis (Hashimoto's disease).
* The classical features are lethargy, delayed deep tendon reflexes, weight gain,
bradycardia, intolerance to cold, hoarse voice, greasy skin and a proximal myopathy.
* Drug metabolism and excretion are reduced, so these patients often exhibit increased
sensitivity to certain drugs including opioids.
77-The following tumour markers are strongly associated with the tumours shown:
1. Alpha-FP and hepatocellular carcinoma.
2. Beta-HCG and choriocarcinoma.
3. CA-19.9 and ovarian carcinoma.
4. CA-125 and pancreatic carcinoma.
5. CEA and colorectal carcinoma.
Teaching Notes for Question 77
Theme: Tumour markers
The ideal tumour marker would be specific to a particular tumour, undetectable if the
tumour was not present, and also sensitive, always being detectable if the tumour was
present and present at a concentration directly proportional to the volume of disease
present.
As no tumour marker fulfils these ideals, compromise must be sought, and they are usually
more useful for monitoring response to treatment or disease recurrence than primary
diagnosis.
CEA is generally associated with colorectal cancer (also may be increased in other GI
malignancies and in smokers).
Alpha-FP is increased in hepatocellular carcinoma and in testicular germline tumours (esp.
teratoma).
Beta-HCG is raised in hydatidiform mole and choriocarcinoma and testicular tumours (esp.
seminoma).
CA-125 is associated with ovarian and CA-19.9 with pancreatic cancer.
78-Arterial thromboses are associated with:1. Behcets syndrome
2. Atrial fibrillation
3. Factor V Leiden mutation
4. Protein C deficiency
5. Polycythaemia
Teaching Notes for Question 78
Theme: Arterial thromboses
Arterial thromboses may be associated with Behcets syndrome, atrial fibrillation and
polycythaemia. Factor V Leiden mutation and protein C deficiency are associated with
venous thromboses.
79-Salbutamol:
72
PEDIATRICS
EXAMINATION REVIEW
C1 esterase inhibitor).
73
PEDIATRICS
EXAMINATION REVIEW
menarche).
* Pubic Hair:
Stage
I
prepubertal
II
sparse, downy hair
III
darker, curlier and spreading
IV
adult type hair but not on medial thighs yet
V
adult
* Male: Genitals
Stage
I
prepubertal
II
scrotum and testes grow and scrotal skin coarsens
III
penis lengthens and testes grow
IV
penis broadens and glans develops. Testes/scrotum darken
V
adult
* Female: Breasts
Stage
I
prepubertal
II
breast bud
III
breast and areola enlarge
IV
breast and areola enlarge to form mound above main breast
V
adult - only nipple projects
* Menarche is either 0 or 1.
4: Asymmetrical breast development is a common phenomenon and just needs
reassurance.
5: A recent BMJ paper looked at a sample of 100 girls to establish the average age of
menarche. No data has been collected in a UK population since the 1950s. The median age
of menarche was 12 years and 11 months. There was no difference related to social class
or ethnicity. This suggests that there has been no decrease in the average menarcheal age
in the past 3 decades. The much talked about drop was from late 19th century (14.75
years) to 1950s (12.8 years).
74
PEDIATRICS
EXAMINATION REVIEW
75
PEDIATRICS
EXAMINATION REVIEW
76
PEDIATRICS
EXAMINATION REVIEW
4. No eye movement when 20 mls cold water is perfused into the external auditory meatus.
5. No motor response from painful stimuli.
Teaching Notes for Question 88
Theme: Brain death
An absent gag reflex must be assessed with bronchial stimulation via a tracheal catheter.
Neuromuscular drugs should have been stopped for at least 24 hours before assesment
can be made.
89-The following are true regarding parathyroid hormone actions:
1. Increases calcium and decreases phosphate reabsorption in kidney
2. Increases osteoclastic activity
3. Increases 1,25 dihyroxy vitamin D3 production in the kidney
4. Decreases calcium and increases phosphate reabsorption in kidney
5. The overall effect is to increase calcium and phosphate level in the blood
Teaching Notes for Question 89
Theme: Parathyroid
The overall effect is to increase calcium amd decrease phosphate level in the blood.
77
PEDIATRICS
EXAMINATION REVIEW
3. Haemorrhage.
4. Exercise.
5. Morphine.
Teaching Notes for Question 91
Theme: Vasopressin
The following stimulate vasopressin secretion:
* Increased plasma osmolality,
* Decreased ECF volume,
* Pain, stress, exercise and standing,
* Nausea and vomiting,
* Angiotensin II, clofibrate, morphine and carbamazepine.
The following decrease vasopressin secretion:
* Decreased plasma osmolality,
* Increased ECF volume,
* Alcohol.
Alcohol also reduces the secretion of oxytocin.
92-Regarding ketamine:
1. Is contra-indicated in reversible airways obstruction
2. Is more emetogenic than thiopentone
3. Has anti-sialogogue properties
4. May be used safely in patients with suspected raised intra-cranial pressure
5. Is a potent respiratory depressant
Teaching Notes for Question 92
Theme: Intravenous anesthetic agents
* Ketamine is a mild respiratory stimulant and a potent bronchodilator, making it a good
agent to be used in asthmatics, either for induction or as a 3rd line treatment of an acute
attack in a ventilated patient.
* It causes more post-op nausea and vomiting than thio and propfol (but less than
etomidate) and cholinergic stimulation leads to increased saliva production.
* It causes increased cerebral blood flow, O2 consumption and ICP and frequently leads to
emergence phenomena.
93-Sickle cell disease :1. Is caused by a mutation on chromosome 16
2. Will give a positive sickle test result at birth
3. Is associated with strokes in 25% of patients by the age of 45 years
4. Is an indication for autologous haemopoietic stem cell transplant
5. Can be co-inherited with alpha or beta-thalassaemia
Teaching Notes for Question 93
Theme: Sickle cell disease
Sickle cell disease is an autosomal recessive condition caused by a single amino acid
mutation in the Beta globin gene on chromosome 11. The alpha globin genes are coded on
chromosome 16. It can be co-inherited with alpha or beta thalassaemia. Clinical effects do
not become apparent until 6 to 12 months of age when haemoglobin switching occurs and
the amount of HbF (fetal haemoglobin) decreases and the amount of HbS (sickle
haemoglobin) increases. This is why the sickle test, which detects the presence of sickle
haemoglobin, is often not positive until 6-9 months of age and cannot be guaranteed until
78
PEDIATRICS
EXAMINATION REVIEW
one year of age. Clinical problems include strokes and these are common, occuring in 25%
of patients with homozygous sickle disease by the age of 45 years. Patients with severe
clinical manifestations of sickle disease have been treated with allogeneic bone marrow
transplantation from an HLA-identical sibling. Autologous bone marrow transplantation is
where the patients own bone marrow is replaced and would not be of benefit in this
condition.
94-The femoral artery:
1. Runs as a single trunk from the inguinal ligament to the lower border of the popliteus
2. Is a direct continuation of the internal iliac
3. In the upper third of the thigh the femoral artery is contained in the femoral triangle
(Scarpas triangle)
4. In the lower third of the thigh, lies in the adductor canal (Hunters canal)
5. The superficial epigastric artery is a branch of this vessel
Teaching Notes for Question 94
Theme: Vascular anatomy- femoral artery
The femoral artery is a direct continuation of the external iliac artery.
The branches of the femoral artery are:
- Superficial epigastric
- Deep external pudendal
- Superficial iliac circumflex
- Muscular
- Superficial external pudendal
- Profunda femoris
- Highest genicular
79
PEDIATRICS
EXAMINATION REVIEW
80
PEDIATRICS
EXAMINATION REVIEW
81
PEDIATRICS
EXAMINATION REVIEW
Nightmares are dream experiences loaded with anxiety or fear, of which the individual has
very little recall. The dream experiences are extremely vivid and usually include themes
involving threats to survival, security, or self esteem. Often there is recurrence of the same
or similar frightening nightmare themes.
-Sleep terrors
* Occurs in non REM sleep
* May be remembered
* Considerable autonomic discharge
* Appreciable vocalisation
* Appreciable body motility
* Last no more than 110 minutes
-Nightmares
* Occur in REM sleep
* Usually vivid
* Some autonomic discharge
* No appreciable vocalisation
* No appreciable body motility
* Can continue into the second half of sleep
In a typical sleep terror, child sits up or gets up with a panicky scream, often rushing to
the door as if trying to escape.
102-Nitric Oxide (NO):
1. Is critical in increasing endothelin (ET-1) levels.
2. Is important in the maintenance of pulmonary vascular flow.
3. Increases cAMP.
4. Operates by inhibiting PDEs.
5. Is the same substance as EDHF.
Teaching Notes for Question 102
Theme: Nitric Oxide (NO):
Nitric Oxide is a critical molecule involved in processes as diverse as vasodilation,
angiogenesis, cell signalling, neurotransmission, immune defence and apoptosis etc. Nitric
oxide is a short-lived molecule (with a half-life of a few seconds). Its synthesis is catalysed
by enzymes known as nitric oxide synthases (NOS). These enzymes convert arginine into
citrulline, producing NO in the process. Oxygen and NADPH are co-factors in this reaction.
There are three isoforms of nitric oxide synthase (NOS): neuronal NOS (nNOS), endothelial
NOS (eNOS) and inducible NOS (iNOS). All three isoforms can be found in a variety of
tissues and cell types. nNOS and eNOS are constitutively expressed in mammalian cells
and synthesise NO in response to increases in intracellular calcium and thus calmodulin
levels. iNOS activity does not depend on intracellular calcium levels, but like the other NOS
enzymes, the binding of calmodulin is essential for iNOS activity. In most cell types iNOS
protein levels are very low or undetectable. However, stimulation of these cells with
cytokines or growth factors, can increase transcription of the iNOS gene, with subsequent
production of high concentrations of NO.
The biological effects of NO are mediated through its reaction with a number of targets
such as haem groups, sulfhydryl groups-to produce nitrosothiols- and iron and zinc
clusters. Such a diverse range of potential targets for NO explains the large number of
systems that utilize it as a regulatory molecule. As a consequence of this, abnormal
regulation or control of NO synthesis is capable of affecting a number of important
biological processes and has been implicated in a variety of diseases. Insufficiency is
believed to contribute to the pathogenesis of diseases such as atherosclerosis,
hypertension and myocardial infarction.
NO is also called endothelium-derived relaxing factor (EDRF)- it mediates the relaxation of
vascular smooth muscle. The NO formed in vascular endothelium diffuses to smooth
muscle cells, and activates guanylyl cyclase producing cGMP, which mediates smooth
muscle relaxation. Importantly, NO modulates endothelin and maintains a low pulmonary
vascular resistance.
NO is produced by a number of immune cells. In particular, activated macrophages
produce high concentrations of NO in response to LPS and interferon-gamma. NO has been
shown to kill cells by disrupting enzymes involved in the Kreb's cycle, DNA synthesis and
mitochondria.
82
PEDIATRICS
EXAMINATION REVIEW
83
PEDIATRICS
EXAMINATION REVIEW
5. Most of the venous drainage of the heart occurs via the the Basian system of veins.
Teaching Notes for Question 105
Theme: Cardiac anatomy
The right atrium is larger in size than the left atrium in normal people. The dominant
coronary artery (CA) is the one that supplies the posterior part of the interventricular
septum. This is supplied by the right CA (RCA) in 85% cases (right dominant circulation)
and by the left circumflex (LCx) in the remaining (left dominant circulation). The sinoatrial
node is almost always supplied by the RCA. Venous drainage from the heart is mainly into
epicardial cardiac veins, draining into the coronary sinus and then into the right atrium.
The 4 pulmonary veins drain into the left atrium.
106-Mitral valve prolapse is associated with:1. Cyanosis
2. Bacterial endocarditis
3. Chest pain
4. A systolic 'click' on auscultation
5. Foetal arrythmias
Teaching Notes for Question 106
Theme: Mitral valve prolapse.
Mitral valve prolapse is a common syndrome and the vast majority of patients are
asymptomatic. A variety of non-specific symptoms may be present including palpitations,
postural hypotension, chest discomfort, and when MR is severe, symptoms of diminished
cardiac reserve predominate. Autonomic dysfunction occurs commonly in MVP and this has
been suggested as the cause for the chest discomfort. The unique physical finding in MVP
is a systolic click at least 0.14sec after S1. The click is often followed by a mid- to latecrescendo systolic murmur that continues to A2. The outlook for MVP in children is
excellent, a large majority remaining asymptomatic for many years. There is an increased
risk of infective endocarditis, although the incidence appears to be extremely low in
patients with a midsystolic click only; the incidence rises in patients with a systolic
murmur. Acute hemiplegia, TIAs, cerebellar infarcts, amaurosis fugax and retinal arteriolar
occlusions all appear to occur more frequently.
107-Normal arterial pH is:
1. Equal to [H+] of 34-46 nmoles/L
2. Lower than intracellular pH
3. Not essential to maintain ion distribution
4. Measured rather than calculated in a pH probe
5. Decreased in pregnancy
Teaching Notes for Question 107
Theme: Acid/base balance
A. pH 7.35-7.45 = [H+] 36-45nmol/L
B. Intracellular pH = extracellular pH - they use this question a lot
C. Normal pH absolutely essential for enzyme function/ion distribution/protein stability
D. pH measured in pH probe
E. In pregnancy, pH increases because of excretion of bicarbonate
108-A 16 year old girl is admitted with progressive weight loss for which no organic cause
can be found. The following factors would be against a diagnosis of anorexia nervosa:-
84
PEDIATRICS
EXAMINATION REVIEW
85
PEDIATRICS
EXAMINATION REVIEW
anticholinergic effects, and a beta-blocker for treatment of SVTs. Other antidotes include
desferrioxamine for iron, calcium EDTA dimercaprol for lead poisoning, dimercaprol for
heavy metal poisoning, ethanol for ethylene glycol, dicobalt edetate for cyanide, digoxinspecific antibody for digoxin, naloxone for opiates, N-acetylcysteine for paracetamol,
Fuller's earth for paraquat, and vitamin K for warfarin.
111-The following are true:
1. The uptake of iodide into the thyroid follicular cell is by active transport
2. T4 has a half-life of 24 hours
3. About 99% of circulating T4 is protein bound
4. T3 is roughly 10 times more potent than T4
5. All steps in thyroid hormone synthesis are stimulated by thyroid peroxidase
Teaching Notes for Question 111
Theme: Thyroid hormones
The main steps are as follows:
1. Active uptake of iodide, which is concentrated to 20 times that of plasma.
2. Oxidation of iodide to iodine, which combines with tyrosine residues to give monoiodotyrosine (MIT) and di-iodotyrosine (DIT).
3. Coupling of 2 DIT residues gives T4 while addition of MIT and DIT gives T3.
*Both T4 and T3 are highly bound to thyroid binding globulin or albumin.
*T4 has a half-life of 7 days T3 is 1 day.
*T3 is 3 to 5 times more potent than T4.
*TSH stimulates all steps in thyroid hormone synthesis - thyroid peroxidase catalyses the
2nd and 3rd steps.
112-Restriction fragment length polymorphisms (RFLPs):1. Are found every 100 bases
2. Are useful in diagnosis of multifactorial diseases
3. Are useful in linked-gene diseases
4. Are generated by digestion of DNA with bacterial enzymes
5. Are detected by Northern blotting
Teaching Notes for Question 112
Theme: Restriction fragment length polymorphisms
The term restriction fragment length polymorphisms (RFLP) refers to the different patterns
seen when DNA from different persons is digested by bacterial enzymes (restriction
enzymes). These enzymes are produces by bacteris (probably as a defence against
viruses) and cut DNA at specific sequences, sequences not found in the bacterium's own
DNA). When human DNA is subjected to digestion, the DNA is cut only where the particular
sequence occurs producing fragments of DNA. These can be resolved according to size by
gel electrophoresis, and then the patterns of different fragment sizes compared from one
sample to another. If a mutation changes the DNA sequence, the enzyme will not cut and
so the sizes of the DNA fragments will be different. RFLPs can be used to link phenotype
(disease) to genotype (sequence), but further detailed analysis of the exact mutation will
still be necessary to identify the gene responsible. Northern blotting is used for the
analysis of RNA not DNA.
113-Which of the following statements are true:
1. Data may be described as qualitative or quantitative
2. Discrete data may include number of children
86
PEDIATRICS
EXAMINATION REVIEW
87
PEDIATRICS
EXAMINATION REVIEW
88
PEDIATRICS
EXAMINATION REVIEW
119-Which of the following are common symptoms associated with raised intracranial
pressure in children?
1. Morning headache
2. Abdominal pain
3. Double vision
4. Ataxia
5. Seizure
Teaching Notes for Question 119
Theme: Benign intracranial hypertension
Benign intracranial hypertension is characterised by increased intracranial pressure in the
absence of a mass lesion or hydrocephalus. It is idiopathic in 50%.
Secondary causes include:
* Drugs (corticosteroids, tetracycline, vitamin A and oral contraceptives);
* SLE;
* Head trauma;
* Otitis media;
* Sinusitis;
* Adrenal insufficiency;
* Hyperthyroidism;
* Hypoparathyroidism;
* Hyperadrenalism;
* Leukaemia;
* Polycythemia;
* Guillain-Barr syndrome;
* Iron deficiency anaemia.
Headache is the commonest feature; others include diplopia, caused by 6th nerve palsy
(abducens). Less commonly, some may have nausea, vomiting, paraesthaesia, neck
stiffness, and ataxia. Papilloedema is a common early finding. If not treated, the
papilloedema will progress and cause optic nerve atrophy. Cranial CT or MRI is essential
with ophthalmology review. Lumbar puncture will help to relieve headache and preserve
the vision. If the pressure is high, it should be reduced by 40% - the normal pressure lies
between 12-16 mmHg. Acetazolamide can be started if patient cannot tolerate second or
third lumbar puncture when symptoms recur. In spite of all these, if a child is still
symptomatic with progressive loss of vision, dexamethasone can be given for two weeks
or/and consider optic nerve fenestration or lumboperitoneal shunt.
120-The following is true regarding endotoxin:1. It is a protein
2. It is heat-stable
3. It is produced by some Gram-positive bacteria
4. All endotoxin has the same physiological effect
5. It is secreted by bacteria during active infection
Teaching Notes for Question 120
Theme: Endotoxin
Endotoxin is composed of lipopolysaccharide in the Gram-negative cell wall. Unlike
exotoxin, the effects of endotoxin are all non-specific. Whereas exotoxin is secreted by
viable bacteria, endotoxin is released only following bacterial cell lysis or death.
89
PEDIATRICS
EXAMINATION REVIEW
90
PEDIATRICS
EXAMINATION REVIEW
91
PEDIATRICS
EXAMINATION REVIEW
92
PEDIATRICS
EXAMINATION REVIEW
93
PEDIATRICS
EXAMINATION REVIEW
94
PEDIATRICS
EXAMINATION REVIEW
lower eyelid (to allow cells to be collected dont forget it is an intracellular organism) by
direct fluorescent antibody, ELISA or PCR. Tetracycline ointment topically is combined with
oral erythromycin the oral antibiotic is to prevent relapse after ointment is discontinued
and to prevent progression to pneumonia. Gonococcal conjunctivitis presents earlier than
chlamydial disease (usually within 2 days), is diagnosed on gram stain and culture and
should be treated with IV penicillin and chloramphenicol eye drops. Dont forget sexual
health screening for the mother and informing public health of ophthalmia neonatorum.
Adenovirus causes conjunctivitis in summer outbreaks; enterovirus, coxsackie and herpes
simplex are other viral causes.
134-The following statements are true:
1. The vagus nerve is responsible for normal bronchial tone
2. There are 18 bronchopulmonary segments
3. Most of the alveoli in the adult lung are present at birth
4. The surface markings of the right middle lobe approximate to the axilla
5. The major component of surfactant is dipalmitoyl lecithin
Teaching Notes for Question 134
Theme: Anatomy
2. There are 5 lobes divided into 18 segments.
3. At birth there are 25 million alveoli, which increase in the adult to 300 million per lung.
4. The right middle lobe extends from the 5th rib in the axilla to the 4th costochondral
junction superiorly and to the 6th costochondral junction inferiorly
135-In beta thalassaemia:1. Beta thalassaemia major presents at birth with profound anaemia
2. The reticulocyte count is low
3. Diagnosis can be made by Haemoglobin electrophoresis
4. Anaemic patients should receive regular iron supplementation
5. Splenectomy is contraindicated due to extramedullary haemopoiesis
Teaching Notes for Question 135
Theme: beta thalasseamia
Beta thalassemia syndromes are a group of hereditary disorders characterized by a genetic
deficiency in the synthesis of beta-globin chains. In the homozygous state, beta
thalassemia (ie, thalassemia major) causes severe transfusion-dependent anemia with
high reticulocyte count.
The anaemia becomes manifest in late infancy. Diagnosis is by haemoglobin
electrophoresis.
Increased iron deposition resulting from multiple life-long transfusions and enhanced iron
absorption results in secondary iron overload. This overload causes clinical problems
similar to those observed with primary hemachromatosis (eg, endocrine dysfunction, liver
dysfunction, cardiac dysfunction)
o The physical findings are related to severe anemia, ineffective erythropoiesis,
extramedullary hematopoiesis, and iron overload resulting from transfusion and increased
iron absorption.
o Skin may show pallor from anemia and jaundice from hyperbilirubinemia.
o The skull and other bones may be deformed secondary to erythroid hyperplasia with
intramedullary expansion and cortical bone thinning.
o Heart examination may reveal findings of cardiac failure and arrhythmia, related to either
severe anemia or iron overload.
o Abdominal examination may reveal changes in the liver, gall bladder, and spleen.
Hepatomegaly related to significant extramedullary hematopoiesis typically is observed.
95
PEDIATRICS
EXAMINATION REVIEW
Patients who have received blood transfusions may have hepatomegaly or chronic hepatitis
due to iron overload; transfusion-associated viral hepatitis resulting in cirrhosis or portal
hypertension also may be seen. The gall bladder may contain bilirubin stones formed as a
result of the patient's life-long hemolytic state. Splenomegaly typically is observed as part
of the extramedullary hematopoiesis or as a hypertrophic response related to the
extravascular hemolysis.
o Extremities may demonstrate skin ulceration.
o Iron overload also may cause endocrine dysfunction, especially affecting the pancreas,
testes, and thyroid.
96
PEDIATRICS
EXAMINATION REVIEW
97
PEDIATRICS
EXAMINATION REVIEW
98
PEDIATRICS
EXAMINATION REVIEW
99
PEDIATRICS
EXAMINATION REVIEW
100
PEDIATRICS
EXAMINATION REVIEW
4. ADA (adenosine deaminase) and PNP (purine nucleotide phosphorylase) deficiency cause
accumulation of nucleotide metabolites that are toxic to developing T-cells resulting in T-cell and
B-cell deficiencies and a SCID phenotype
5. SCID can be due to an autosomally inherited defect in DNA repair analogous to the genetic
defect in the SCID mouse strain
Teaching Notes for Question 147
Theme: Severe combined immunodeficiency
Patients with SCID always have deficient T-cells but may or may not have absent B-cells.
Several defects can lead to the phenotype of severe combined immunodeficiency. In Xlinked SCID there is a mutation in the gene encoding the common gamma chain a chain
shared by receptors for a number of interleukin cytokines -(IL-2, 4, 7, 9 and 15). There is
thus impaired T-cell differentiation as the interleukins are important T-cell growth factors.
In this type of SCID, B-cells are present (T-B+ SCID). Humoral immunodeficiency occurs
because of a lack of T-cell help. T-B+ SCIDs may also be due to an autosomal recessive
mutation in Jak 3 an intracellular enzyme which is a component of the signalling pathway
induced by various cytokines.
T-B- SCIDs are due to adenosine deaminase (ADA) deficiency, purine nucleoside
phosphorylase (PNP) deficiency, or RAG-1 or 2 deficiency, all of which are autosomal
recessive. ADA and PNP are enzymes which catalyze the metabolism of purines via the uric
acid pathway. Deficiency of either of these enzymes leads to the accumulation of toxic
metabolites which severely affect developing T-cells. B-cell development is also affected to
a lesser extent. Functional Rag proteins are required for initiation of recombinatorial
rearrangement of the genes encoding the antigen-specific receptors of both T- and B-cells.
Patients with mutations resulting in non-functional Rag fail to develop both B- and Tlymphocytes. Other known causes of T-B- SCID include mutations of DNA-dependent
protein kinase (DNA-PK - an enzyme involved in DNA repair) and lack of expression of MHC
class II or MHC class I molecules (bare lymphocyte syndrome).
Patients with SCID are susceptible to infection with bacteria and viruses as there is a
complete defect in adaptive immunity.
148-Lower esophageal sphincter tone is decreased by:
1. Cisapride
2. Progesterone
3. Suxamethonium
4. Ganglion blockers
5. Ondansetron
Teaching Notes for Question 148
Theme: Esophageal sphincters
The effect of progesterone on oesophageal sphinter tone is one of the many reasons for
the increase in gastro oesophageal reflux in pregnancy.
149-A normally developing 18-month-old child:
1. Should recognise his/her name
2. Can easily distinguish primary colours by name
3. Plays in parallel with his/her peers
4. Should imitate observed actions
5. Should be able to copy a vertical line
Teaching Notes for Question 149
Theme: Developmental milestones
18-month-old child - normal developmental milestones
101
PEDIATRICS
EXAMINATION REVIEW
102
PEDIATRICS
EXAMINATION REVIEW
103
PEDIATRICS
EXAMINATION REVIEW
104
PEDIATRICS
EXAMINATION REVIEW
105
PEDIATRICS
EXAMINATION REVIEW
106
PEDIATRICS
EXAMINATION REVIEW
107
PEDIATRICS
EXAMINATION REVIEW
* Its oral bioavailability is only 1 - 2%, it does not cross the BBB or the placenta owing to
its highly ionized state at physiological pH and also its quaternary amine structure.
* It is metabolized by plasma and hepatic esterases and is excreted predominantly in the
urine, but also approx 30% in the bile.
165-Recognized associations of systemic conditions and cataracts include:
1. Muscular dystrophy
2. Galactosemia
3. Hyperthyroidism
4. Relapsing polychondritis
5. Septicemia
Teaching Notes for Question 165
Theme: Cataracts, systemic diseases
A cataract is simply any loss of the transparency of the lens of the eye. The list of systemic
associations with cataracts is too long to discuss in full, but important associations include:
*
*
*
*
*
*
*
*
*
*
166-The following statements are true:1. A moro reflex with the fists remaining clenched is normal.
2. The plantar grasp persists longer than the palmar grasp
3. Deep tendon reflexes are difficult to elicit at birth
4. An extensor Babinski response is normal in the first year
5. A newborn baby from 32 weeks gestation will turn towards a light
Teaching Notes for Question 166
Theme: neuro-developmental assessment
Most primitive reflexes disappear by 6 months of age. Exceptions are the plantar reflex (9
months), asymmetric tonic neck reflex (9 months), stepping reflex (11 months). The moro
reflex consists of arm extension followed by flexion with extension of the fingers, and
usually flexion of the thighs at the hips. A moro reflex with the fists remaining clenched is
abnormal. Deep tendon reflexes are present and usually brisk at birth. The Babinski
response is usually extensor until the infant begins to walk (usually from 1 year). A
newborn baby from 32 weeks gestation with a normally functioning visual cortex will turn
the head towards a light.
167-Glucocorticoids:
1. Are secreted from the zona fasciculata within the adrenal cortex.
2. Molecules consist of 21 carbon atoms.
3. Cause retention of sodium.
108
PEDIATRICS
EXAMINATION REVIEW
4. Inhibit gluconeogenesis.
5. Reduce lymphocyte counts.
Teaching Notes for Question 167
Theme: Glucocorticoids
The glucocorticoids consist of cortisol and corticosterone. The main effects are in protein,
glucose and fat metabolism. There is a slight mineralocorticoid effect, and also an antiinflammatory effect.
168-In the treatment of attention deficit hyperactivity disorder :1. Behavioural therapy is never useful
2. Pharmacological intervention includes the use of stimulant drugs such as amphetamines
3. Methylphenidate (Ritalin) should not be given later than 4pm
4. Parenting techniques can be taught
5. Poor appetite is a common side affect of methylphenidate (Ritalin)
Teaching Notes for Question 168
Theme: Attention deficit hyperactivity disorder
Treatment of attention deficit hyperactivity disorder: stimulant medicationmethylphenidate (Ritalin), dexamphetamine. Not to be taken after 4pm.
Side effects include difficulty sleeping, loss of appetite, abdominal pain, headache,
thrombocytopenia.
Behavioural modification, parenting techniques.
Outcome: symptoms tend to lessen as child gets older.
169-Concerning the synthesis of thyroid hormones:
1. TRH is released by the anterior pituitary.
2. Iodine is taken up passively by the thyroid gland.
3. Thyroid peroxidase catalyses the formation of thyroxine.
4. T4 is released bound to thryroglobulin.
5. T4 is released in greater quantities than T3.
109
PEDIATRICS
EXAMINATION REVIEW
5. Pyloric stenosis
Teaching Notes for Question 170
Theme: Autosomal recessive diseases
Diseases that are inherited in an autosomal recessive manner are usually those in which
there is a biochemical defect. Primary ciliary dyskinesia (PCD) is an exception since at least
one of the genes encodes a structural protein of the cilia, dynein. The inheritance of pyloric
stenosis and insulin dependant diabetes is multifactorial although it has been proposed
that pyloric stenosis is inherited as autosomal dominant with reduced penetrance on a
multifactorial background.Batten Disease is an AR disorder also known as Spielmeyer-VogtSjogren-Batten Disease, it is the most common form of a group of disorders called
Neuronal Ceroid Lipofuscinoses (or NCLs). Affected children suffer progressive mental
impairment, worsening seizures, and progressive loss of sight and motor skills.
171-Noradrenaline
1. Is a neurotransmitter at the pre-ganglionic sympathetic nervous system
2. Is produced by the adrenal cortex
3. May cause a reflex bradycardia when given by infusion
4. Causes a rise in metabolic rate
5. It acts predominantly via the beta adrenoceptor
Noradrenaline is a catecholamine produced by the
adrenal medulla. It also acts as a neurotransmitter
in the post-ganglionic sympathetic neurons and
parts of the brain. Its actions are mediated mostly
through the alpha-adrenoceptor, though it does have
some action at the beta-adrenoceptor. It causes vasoconstriction, leading to a rise in blood
pressure. It activates the baroreceptor reflex, and hence may cause a reflex bradycardia.
It causes an increase in metabolic rate, hence, in the heart, although coronary perfusion
may be increased, myocardial oxygen consumption will also be increased.
Teaching Notes for Question 171
Theme: Catecholamines
172-In a healthy baby the transition from fetal to neonatal circulation involves:1. Functional closure of the foramen ovale in the first 24 hours
2. Blood flow in the ductus arteriosus continues from right to left until its closure
3. Decrease in pulmonary artery resistance following closure of the ductus arteriosus
4. The ductus arteriosus closes in response to decreased oxygen concentrations
5. The umbilical artery is a branch of the common iliac artery
Teaching Notes for Question 172
Theme: Cardio-pulmonary changes at birth
Functional closure of the ductus arteriosus occurs soon after birth but anatomical closure
can take upto one week. As pulmonary pressures fall after birth, blood flow in the ductus is
reversed ie from left to right. The umbilical artery is a branch of the internal iliac artery.
Factors influencing closure of the ductus include:1. Increased oxygen concentrations
2. Decreased prostaglandin levels
3. Drop in pulmonary artery pressures
N.B. Prostaglandin E2 keeps the ductus open.
173-The pancreas:
1. Is entirely retroperitoneal
110
PEDIATRICS
EXAMINATION REVIEW
111
PEDIATRICS
EXAMINATION REVIEW
112
PEDIATRICS
EXAMINATION REVIEW
113
PEDIATRICS
EXAMINATION REVIEW
114
PEDIATRICS
EXAMINATION REVIEW
115
PEDIATRICS
EXAMINATION REVIEW
116
PEDIATRICS
EXAMINATION REVIEW
Phagocytic cells enter tissues by rolling on selectins and binding via integrins to
ICAM (and similar molecules). They transmigrate by binding to PECAM and similar
adhesion molecules.
188-With regards to aniridia:1. Two thirds of the cases are sporadic.
2. It is usually bilateral.
3. Vision is commonly decreased.
4. It is associated with partial deletion of long arm of chromosome 11.
5. Wilm's tumour is associated with familial aniridia.
117
PEDIATRICS
EXAMINATION REVIEW
118
PEDIATRICS
EXAMINATION REVIEW
119
PEDIATRICS
EXAMINATION REVIEW
2. Hypothyroidism.
3. Haemolytic disease.
4. Cytomegalovirus infection.
5. Cystic fibrosis.
120
PEDIATRICS
EXAMINATION REVIEW
121
PEDIATRICS
EXAMINATION REVIEW
4. SLE.
5. Hyperthyroidism.
122
PEDIATRICS
EXAMINATION REVIEW
Questions
(PART 2)
123
PEDIATRICS
EXAMINATION REVIEW
1- A 14-year-old boy is rescued from a burning building and is brought to the hospital. He
has nausea, vomiting, diarrhoea and abdominal pain. He is confused. The carboxyhaemoglobin
(COHb) level is 25%.
3-Choose the correct matching pair of disease and antibody out of the following options:Options:
A. Anti-liver/kidney microsomal antibodies (anti LKM-1) in type I autoimmune hepatitis
B. Anti-smooth muscle antibodies in Primary biliary cirrhosis
C. Anti-nuclear antibodies in Primary biliary cirrhosis
D. Anti-mitochondrial antibodies in type II autoimmune hepatitis
E. Anti liver cytosol antibodies in primary sclerosing cholangitis
4-hich one of the following cells in the lung parenchyma produces surfactant?
Options:
A. Alveolar macrophage
B. Endothelial cell
C. Goblet cell
D. Type I pneumocyte
E. Type II pneumocyte
124
PEDIATRICS
EXAMINATION REVIEW
5-Which of the following conditions with renal involvement is the most likely to
have normal complement levels:Options:
A. Grade IV lupus nephritis
B. Post-streptococcal glomerulonephritis
C. Subacute bacterial endocarditis
D. Type II cryoglobulinaemia
E. IgA nephropathy (Buergers disease)
7-The best treatment option for a patient with Usual Interstitial Pneumonia (UIP or CFA
(Cryptogenic fibrosisng alveolitis) is :Options:
A. Lung transplantation
B. Interferon-1-Beta
C. Azathioprine
D. Cyclophophamide
E. High -dose steroids
125
PEDIATRICS
EXAMINATION REVIEW
8-What is the diagnosis that can you infer from the X-ray abdomen?
Options:
A. LBO (Large bowel obstruction)
B. SBO (Small bowel obstruction)
C. Toxic Megacolon
D. Chagas Disease
E. Ankylosing Spondylitis
9-A 6 month old boy with severe eczema presents with easy bruising and two nose bleeds.
He is below the 2nd centile for weight. He has had 3 previous admissions to hospital with
pneumonia and has required two courses of antibiotics for otitis media.
What is the most likely underlying diagnosis :Options:
A. Primary antibody deficiency
B. Ataxia telangiectasia
C. Adenosine deaminase deficiency
D. C4 deficiency
E. Wiskott Aldrich syndrome
126
PEDIATRICS
EXAMINATION REVIEW
13-A 9-year-old has a cranial CT scan. The radiologist reports the presence of intracranial
calcification.
Which of the following diagnoses is least likely to be associated with the radiologists
report?
Options:
A. Subdural haematoma
B. Sturge-Weber syndrome
C. Toxoplasmosis
D. Ataxia telangiectasia
E. Hypoparathyroidism
127
PEDIATRICS
EXAMINATION REVIEW
16-A girl with an unusual facial appearance is found to have coarctation. Possible diagnoses
are:
Options:
A. Noonan's syndrome
B. William's syndrome
C. Down's Syndrome
D. Turner's syndrome
E. Edward's syndrome
128
PEDIATRICS
EXAMINATION REVIEW
21-Which of the following renal disorders is associated with normal serum complement
levels?
Options:
A. Membranoproliferative glomerulonephritis
B. All forms of lupus nephritis
C. Shunt nephritis
D. Post-infectious glomerulonephritis
E. Goodpastures disease
129
PEDIATRICS
EXAMINATION REVIEW
22-What is the most likely explanation for these biochemical changes in a 15-year-old girl
with erosive changes in her mouth? Na 138 mmol/l K 3.2 mmol/l HCO3 36 mmol/l Ca
2.4 mmol/l P04 1.0 mmol/l Urine chloride 10 mmol/l
Options:
A. Primary hyperparathyroidism
B. Primary hyperaldosteronism
C. Persistent vomiting
D. Anorexia nervosa
E. Ehlers Danlos syndrome
23-A patient presents with acute infective arthritis. The resulting inflammation:
Options:
A. Causes the release of C8a, a potent chemokine
B. Causes the release of CRP, the half-life of which is several days
C. May be defined as increased vascular permeability accompanied by an infiltration of inflammatory
cells
D. Is always harmful
E. Once initiated, it follows a stereotypical cascade, which is not regulated
24-What does this ECG of an 8-year-old boy with Ebsteins anomaly show?
Options:
A. Atrial flutter
B. Supraventricular tachycardia
C. Sinus tachycardia
D. AV reentrant tachycardia due to accessory pathway
E. Ectopic atrial tachycardia
130
PEDIATRICS
EXAMINATION REVIEW
131
PEDIATRICS
EXAMINATION REVIEW
31-A 17 year old girl presents with primary amenorrhoea. She has recently started
developing breasts. She is 145 cm tall and weighs 48 kg. Both her sisters (age 14 and 19)
are greater then 170cm tall and are well.
Hb 10.5 ,
ALP 320
MCV 108
Bili 12,
WCC 7.6 ,
AST 34
Ca2+ 1.9
U&E normal,
PO4 0.7 ,
TFTs normal
What is the underlying diagnosis ?
Options:
A. Crohns disease
B. Coeliac disease
C. Cystic fibrosis
D. Addisons disease
E. Ulcerative colitis
132
PEDIATRICS
EXAMINATION REVIEW
34-The following statements about techniques using FISH (fluorescent in situ hybridisation)
are true except:
Options:
A. The technique can be performed on uncultured cells
B. The technique requires specialised microscopes
C. The technique can detect small deletions in DNA
D. The technique is used instead of light microscopy for routine chromosome analysis
E. The technique can be used to map the position of genes on a chromosome
133
PEDIATRICS
EXAMINATION REVIEW
38-In the investigation of a patient who is admitted following 3 dizzy spells each markedly
improved by eating:Options:
A. Urine sulphonylurea levels are unhelpful
B. A 72hr fast should be organised, with the doctor called when the blood sugar reaches 4mmol/l.
C. Serum insulin, glucose and C-peptide should be taken every morning for 7 days
D. Serum C-peptide will not be raised in exogenous insulin use
E. A family history of scars in the neck is irrelevant
39-A 16-year-old girl presents with fever, joint aches and a red rash on her face, neck and
hands. She attributed the rash to severe sunburn and says that she has sensitive skin. What
are the most appropriate tests to request?
Options:
A. Anti nuclear antibody and c-reactive protein
B. c-reactive protein and erythrocyte sedimentation rate
C. Full blood count and chemistry
D. Anti-nuclear antibody and erythrocyte sedimentation rate
E. Complement levels
134
PEDIATRICS
EXAMINATION REVIEW
42-An 18-year-old man is referred to the outpatient clinic as his general practitioner has
found him to have microscopic haematuria. He also has bilateral sensorineural deafness
with a sister and younger brother who are similarly afflicted.
What is the most useful investigation?
Options:
A. Urine microscopy
B. Serum creatinine
C. Anti-GBM antibodies
D. Audiometry
E. Renal biopsy
135
PEDIATRICS
EXAMINATION REVIEW
136
PEDIATRICS
EXAMINATION REVIEW
47-Which of the following statements correctly describes the clinical phenotypes which are
associated with a deficiency of the various complement components?
Options:
A. C5-C9 deficiency and mycoplasma infection
B. C1q deficiency and angio-oedema
C. C2 deficiency and rheumatoid arthritis
D. C3 def is associated with partial lipodystrophy
E. A raised C4 and low C3 in SLE is associated with active disease
137
PEDIATRICS
EXAMINATION REVIEW
49-All of the following are clinical features of severe aortic stenosis except:
Options:
A. Fourth heart sound
B. Third heart sound
C. Soft aortic component of second heart sound
D. Systolic thrill
E. Length of murmur in systole
50-A two-year-old boy is brought to the resuscitation room. He has been unwell for 12 hours
with fever and has vomited once. On examination, his airway is patent and he is breathing
oxygen via a mask with non-rebreathing bag with oxygen saturations of 100%. His heart
rate is 160/min, capillary refill time is 5 seconds and he has a purpuric rash over his lower
limbs. He is not responsive to pain. A colleague has been trying for 5 minutes to secure
intravenous access when you arrive. The next step should be:
Options:
A. Continue to try for IV access and then give 20mL/kg fluid if successful
B. Check blood sugar
C. Secure intraosseus access and give 20mL/kg fluid
D. Give immediate IM ceftriaxone before securing IV access
E. Check a capillary blood gas
51-A patient is noted to have a small left pupil. There is also a ptosis present in this eye. His
mother has noted that the left side of his face is dryer than normal. Where is the lesion
likely to be?
Options
A. Oculomotor nerve
B. Trigeminal nerve
C. Facial nerve
D. Vagus nerve
E. Cervical sympathetic trunk
138
PEDIATRICS
EXAMINATION REVIEW
52-A patient is diagnosed as having a Horner's syndrome following carotid surgery. Which of
the following is NOT a symptom of Horners syndrome?
Options:
A. Pupillary constriction
B. Ptosis
C. Sinking in of one eye
D. Absence of sweating on face and neck
E. Lack of lacrimation
54-The lipid-lowering effect of a new statin is compared to atorvastatin. Critical level is set
at 0.01 and difference is found (p = 0.001).
What is the best conclusion to draw?
Options
A. A larger sample should have been used
B. The alternative hypothesis should be accepted
C. The probability of a type I error occuring is 1%
D. The p-value associated with this statin means that it is a clinically significant new drug
E. The trial would have been better run against placebo
139
PEDIATRICS
EXAMINATION REVIEW
56-The following statements are true about Tetralogy of Fallot (TOF) except:
Options:
A. Children with TOF are susceptible to cerebral abscess
B. The chest X-ray shows oligaemic lung fields
C. The second heart sound is soft and single
D. TOF is associated with dextroposition of the aorta
E. This disorder results in a left to right shunt
57-In terms of the NICE guideline of screening for diabetic nephropathy in Type 2 patients:
Options:
A. The target HbA1c is about 8%.
B. A renal consultation is advised if the serum creatinine rises above 250 mol/l.
C. All patients with confirmed microalbuminuria, if no contraindications, should be on an ACE inhibitor.
D. Blood pressure is less important than reducing the degree of proteinuria.
E. Calculating ratios of albumin and creatinine in the urine is not as useful as the total urine protein
level.
140
PEDIATRICS
EXAMINATION REVIEW
60-Following a dislocated shoulder in a rugby match that gets out of hand, a patients
axillary nerve is damaged. What is the likely result?
Options:
A. The teres major atrophies.
B. The rounded contour of the shoulder disappears.
C. A loss of sensation may occur in the lateral forearm.
D. The patient may lose the ability to adduct the arm.
E. The patient may have a wrist drop.
61-Steve, a 10-year-old boy presents with a chronic cough, recurrent lung infections and
clinical evidence of obstructive pulmonary disease. His mother reveals that when he was 2
years old he was diagnosed as having malabsorption, and she has noticed that his sweat
tastes excessively salty.
All the above are features of this condition EXCEPT:
Options:
A. 510% of cases manifest with meconium ileus at birth
B. In 70% of patients, the genetic abnormality is a four-base pair deletion at the locus on chromosome
7
C. Pancreatic abnormalities are present in 80% of patients
D. The primary defect is in the transport of chloride ions across epithelial tissue
E. Staphylococcus aureus and pseudomonas aeruginosa are two pathogens most commonly isolated
from the sputum of patients with this condition
141
PEDIATRICS
EXAMINATION REVIEW
62-A 26-year-old airhostess is brought to the A&E department following a witnessed grand
mal seizure.
She has been previously fit and well except for a history of mild intermittent arthralgia
affecting both hands. Her only medication is the OCP. She smokes 20 cigarettes per day and
drinks 15 units of alcohol per week.
* On examination she has no focal neurology.
* There is slight puffiness of her fingers and she has low-grade pyrexia of 37.6C.
* BP 160/98 mmHg.
* Fundoscopy reveals silver wiring and AV nipping bilaterally.
* Urinalysis tests 3+, blood 2+ proteins.
Her blood test results reveal:
Hb
9.9 g/dl
MCV
83 fl
WCC
3.1 x 109/l
Platelets
117 x 109/l
CRP
2 g/l
ESR
71 mm in the first hour
Sodium
134 mmol/l
Potassium
5.1 mmol/l
Urea
10.1 mmol/l
Creatinine
178 mmol/l
Albumin
30 g/l
Total protein
80 g/l
LFTs
Normal
BM
7.8 mmol
Clotting screen:
PTT
APTT
VDRL
11 seconds
39 seconds
Positive 1:320
Negative
Not detected
63-An elderly man presents to casualty with a wrist drop and pain radiating down his right
arm. He recently visited his GP for a deep IM steroid injection. The probable site of the
injection was the:
Options:
A. Posterior cord of the brachial plexus.
B. The medial cord of the brachial plexus.
C. The long thoracic nerve.
D. The C5 spinal root.
E. The ulnar nerve.
64-A boy presents with fever, arthralgia and headache and a red, annular rash in his axilla,
142
PEDIATRICS
EXAMINATION REVIEW
which his mother says has been present for one week and has increased in size. Which is
the most likely diagnosis?
Options:
A. Kawasaki disease
B. Infectious mononucleosis
C. Scarlet fever
D. Lyme disease
E. Rheumatic fever
66-A 2-year old boy is admitted with fever. The presence of jaundice would make which one
of the following diagnoses more likely?
Options:
A. Typhoid
B. Malaria
C. Toxocariasis
D. Lyme disease
E. Cat scratch fever
67-A two-month-old child had his first DTP injection 3 days previously and his mother has
returned because his thigh is swollen. On examination, most of the anterolateral surface of
the thigh is red and indurated. What will you recommend for this child in the future?
Options
A. He should not have any further immunisations
B. He should have all immunisations except pertussis
C. He should receive the acellular pertussis vaccine when next immunised
D. He should receive the oral pertussis vaccine when next immunised
E. He should receive the whole cell pertussis vaccine (as DTP) as usual at his next immunisation
143
PEDIATRICS
*
*
*
*
EXAMINATION REVIEW
144
PEDIATRICS
EXAMINATION REVIEW
Options:
A. Can be precipitated by paracetamol
B. Can produce hyponatraemia due to Diabetes Insipidus
C. Are a feature of porphyria cutanea tarda
D. Are precipitated by the contraceptive pill
E. Are associated with malabsorption
73-A 17-yr-old male medical student presents with a sudden onset of tunnel vision. On
examination, he reports having only central vision over an area of approximately 1 m by 1
m. He reports this at a distance of 3 m and also at 6 m.
Which of the following diagnoses best fits his symptoms?
Options:
A. Optic neuritis
B. Anterior uveitis
C. Methanol poisoning
D. Non-organic symptoms
E. Retinitis pigmentosa
74-A middle aged lady presents to Casualty with a wrist drop and pain radiating down her
right arm after sustaining an injury to her neck, right arm and shoulder during a game of
tennis. The probable site of the injury was the:Options:
A. Posterior cord of the brachial plexus
B. Medial cord of the brachial plexus
C. T1 spinal root
D. C5 spinal root
E. Ulnar nerve
145
PEDIATRICS
EXAMINATION REVIEW
75-A 26-year-old air hostess is brought to A&E following a witnessed grand mal seizure. She
has been previously fit and well except for a history of mild intermittent arthralgia affecting
both hands. Her only medication is the OCP, she smokes 20/day and drinks 15 units of
alcohol per week.
On examination she has no focal neurology. There is slight puffiness of the fingers and low-grade
pyrexia of 37.6C. BP 160/98 mmHg. Fundoscopy reveals silver wiring and AV nipping bilaterally. Urine
analysis tests show 3+ blood, 2+ protein.
Results:
Hb
MCV
WCC
platelets
CRP
ESR
Sodium
potassium
urea
creatinine
Albumin
total protein
LFTs
BM
Clotting screen:
PTT
APTT
VDRL
Drug screen of urine
Plasma alcohol
9.9 g/dl
83 fl
3.1 X 109/l
117 X 109/l
2 g/l
71 mm in the first hour
134 mmol/l
5.1 mmol/l
10.1 mmol/l
178 mmol/l
30 g/l
80 g/l
normal
7.8 mmol
11 seconds
39 seconds
positive 1:320
negative
not detected
The patient then develops acute dyspnoea and examination reveals decreased air entry at the right
base, tachycardia, BP 105/55. O2 sats 94% on room air.
She responds well to prednisolone and azathioprine, but at an OPD appointment 3 months later is
complaining of worsening right hip pain poorly responsive to NSAIDs.
X-ray is normal but bone scan shows increased uptake in the hip.
What is the most likely cause of her hip pain?
Options:
A. Osteoporosis
B. Avascular necrosis
C. Septic arthritis
D. Stress fracture
E. Trochanteric bursitis
76-A 35-year-old woman who complained of easy bruising, weight gain and striae is being
worked up for Cushings syndrome. Her 24-hour urinary free cortisol is 450 nmol. Which is
the next investigation to confirm the diagnosis?
Options:
A. Low-dose dexamethasone suppression test
B. High-dose dexamethasone suppression test
C. Measuring cortisol levels
D. Measuring ACTH levels
E. CRH stimulation test
146
PEDIATRICS
EXAMINATION REVIEW
77-A 26-year-old woman is admitted to hospital for cadaveric renal transplantation. The
operation is uneventful however she is admitted to the Intensive Therapy unit for 48 hours
postoperatively. She is given treatment with prednisolone and cyclosporin, however 10 days
after admission she develops a fever, diarrhoea, headaches and a non-productive cough.
Examination reveals no focal signs of infection. Blood cultures are taken and a CXR is
unremarkable. She is treated with cefotaxime, however after 48 hours the fevers continue
and a repeat CXR shows left lower lobe consolidation.
Options:
A. Streptococcus pneumoniae
B. Klebsiella pneumoniae
C. Pneumocystis carinii
D. Cytomegalovirus
E. Cryptococcus neoformans
78-A 6 week old male baby presents with a 2 day history of multiple seizures. He has been
vomiting non-billious breast milk for the last 3 weeks. On examination the child is afebrile
and dehydrated with a depressed anterior fontanelle. Ultrasound of the abdomen shows
hypertrophic pyloric stenosis. The intravenous (iv) fluid of choice for administration would
be:
Options:
A. iv 50% calcium gluconate
B. iv isolyte P
C. iv 5% dextrose
D. iv sodium bicarbonate
E. iv Hypertonic saline
80-A 15 year old boy is admitted with pneumonia. Investigations show hyponatraemia and
mildly deranged LFTs. CXR shows hazy shadowing in the right mid and lower zones. Therapy
is started and 5 days later he becomes acutely jaundiced with red discolouration of the
urine. Which of the following drugs is the likely cause :Options:
A. Cefotaxime
B. Penicillin
C. Rifampicin
D. Ciprofloxacin
E. Tetracycline
81-Seven weeks after returning from rural Ghana, a 16-year-old boy develops fever and
147
PEDIATRICS
EXAMINATION REVIEW
rigors.
The least useful test is:
Options:
A. Thick and thin blood film
B. IgM for viral haemorrhagic fever
C. Chest radiograph
D. Blood cultures
E. HIV antibody test
82-A 15-year-old is undergoing investigation for hirsuitism. Which of the following would
not be implicated in the aetiology?
Options:
A. Cyclosporin A.
B. Phenytoin.
C. Moxonidine.
D. Congenital adrenal hyperplasia.
E. Cushing's syndrome.
83-A family attends genetic counselling. They have one child affected with Downs syndrome
and are considering another pregnancy.
Which of the following statements is most accurate?
Options:
A. Both parents should have karyotype analysis, but there is no need to karyotye the affected child
B. Most children with Down's syndrome are born to women over the age of 35 years
C. Amniocentesis should only be offered to confirm the diagnosis of Downs syndrome if the family will
consider termination of the pregnancy
D. A karyotype of 46XX excludes a diagnosis of Downs syndrome
E. A nuchal scan together with bHCG and PAP (pregnancy associated protein) between 11 and 13
weeks of pregnancy gives an accurate risk of Downs syndrome in >80% of cases
84-A 17 year old student develops a deterioration in vision in her left eye over three days.
She complains of discomfort in the eye and thinks that difficulty with perception of colour
was the first problem that she noticed, during a trip to an art gallery. On examination, visual
acuity on the left is down to light perception. The pupil appears dilated and does not
constrict to light, although does when a torch is shone in the right eye.
What is the most likely diagnosis :Options:
A. CMV retinitis
B. Optic neuritis
C. Anterior iscaemic optic neuropathy
D. Amaurosis Fugax
E. Central retinal artery occlusion
148
PEDIATRICS
EXAMINATION REVIEW
85-A 29 year old Cypriot man gives a history of recurrent pain and swelling of the left knee.
He also describes recurrent episodes of fever, pleuritic pain, and rash. He is then
hospitalised with severe headache, photophobia and neck stiffness. He is lucid with a GCS of
15.
His blood test results show:
Hb 13.8g/dl
plats 210 X 109/l
WCC 4.9 X 109/l
plasma glucose 6.5 mmol/l
CRP 39g/l
ESR 56 mm in the first hour
RF, ANA, ENA and ANCA negative
CSF analysis:
opening pressure 14cmH2O
protein 0.6 g/l
RBC 2 /ml
WBC 21/ml (100% lymphocytes)
CSF glucose 4.1 mmol/l
no organisms seen on microscopy
48 hour culture of CSF negative
What is the diagnosis?
Options:
A. Familial Mediterranean fever
B. Adult onset Still's disease
C. Behcets syndrome
D. Familial Hibernian fever
E. Non Hodgkins lymphoma
86-Which of the following do not tend to make the symptoms of myasthenia gravis worse?
Options:
A. Prednisolone
B. Aminoglycosides
C. Pyridostigmine
D. Lithium
E. Quinidine
149
PEDIATRICS
EXAMINATION REVIEW
89-In a patient that has been previously diagnosed with Wilsons disease, which of the
following statements is true :Options:
A. Serum cerruloplasmin levels are raised
B. When penicillamine is administrated then urinary copper level rise
C. Is inherited as an autosomial dominant trait
D. May result in retinal abnormalities
E. Most common presentation is with a manic depressive illness
150
PEDIATRICS
EXAMINATION REVIEW
91-A child has had his chromosomes analysed in the course of various investigations and
the result is: 46 XY, t (2;5)(q35;p21.3}.
Which of the following statements is correct?:Options:
A. He has more than 46 chromosomes
B. The result shows all his genetic defects
C. There is a translocation between the short arm of chromosome 2 and the short arm of chromosome
5
D. He is likely to be infertile
E. There is an increased risk of him having a child with difficulties
93-A 9 months old baby girl is brought to you with failure to thrive and rickets. Her parents
tell you that she has always been a very thirsty child, drinking 3-4 bottles of water through
the night, and having soaking wet napping, urine dipstick shows 1+ glucose, and 1+protein.
Which of the following features are you likkely to find on examination and investigation.
Options:
A. Cataract.
B. Corneal opacities.
C. Hyperkalaemia.
D. Metabolic acidosis with a wide anion gap.
E. Nephrocalcinosis.
151
PEDIATRICS
EXAMINATION REVIEW
96-The mother of a baby in the postnatal ward has an antenatal screening test for sickle
haemoglobin. This is positive deposite a normal Hb & MCV. The Hb-electrophoresis shows
HbA 55% and Hbs 45%. Her partner has a microcytic anaemia, and Hb-electrophoresis
shows an HbA band only with HbA2 quantitation of 4.8%.
Which diagnosis is impossible in the baby?
Options:
A. Sickle cell trait.
B. Haematologically normal.
C. B-thalassaemia trait.
D. Sickle-B-thalassaemia.
E. Sickle cell diseas(HbSS).
152
PEDIATRICS
EXAMINATION REVIEW
97-A 9 years old male has always wet the bed, he has no daytime enuresis, which one of
the following statement(s) regarding enuresis is not true?
Options:
A. Runs in families.
B. There is a 30% chance that his father was a bed-wetter.
C. Drinking tea causes bed-wetting
D. At 15 years age, there is a 1% chance that he will still bed-wet.
E. If a child has tried many treatments over a number of years the prognosis is poor.
98-A 18 years old student becomes un well with a sore throat, low grade fever and has some
cervical lymphadenopathy,
on consulting his GP 3 days after the onset of this illness, examination is otherwise
unremarkable, investigations reveals:
FBC
normal ,Na=137
K=4.4 urea=10.1 ,creatinin =141 ,C3= 1.31g/l (0.551.20)C4= 0.42g/l (0.20-0.50),CH50= 87% (50-110) Urinalysis: blood 3+, protein +, few red
cell casts.
What is the most likely diagnosis?
Options:
A. post-streptococcal glomerulonephritis.
B. Wegeners granulomatosis.
C. Ig A nephropathy.
D. Interstitial nephrosis.
E. Henoch-Schonlein purpura.
99-An adolescent male is admitted with progressive limb weakness, on examination he has
flacid weakness predominantly in the legs and absent reflexes. CSF examination shows
markedly raised protein & a mild lymphocytosis,
Which of the following does not imply a worse prognosis?
Options:
A. History of diarrhea in the preceding few weeks.
B. presence of postural hypotension.
C. Markedly reduced FVC.
D. Evidence demyelination on nerve conduction studies.
E. Sever weakness.
153
PEDIATRICS
EXAMINATION REVIEW
1- A 14-year-old boy is rescued from a burning building and is brought to the hospital. He
154
PEDIATRICS
EXAMINATION REVIEW
has nausea, vomiting, diarrhoea and abdominal pain. He is confused. The carboxyhaemoglobin
(COHb) level is 25%.
A
Teaching Notes for Question 1
Theme: Carbon monoxide poisoning
Carboxyhaemoglobin (COHb) levels do not correlate well with the clinical picture. The
indications for hyperbaric oxygen are neurological or psychiatric symptoms, cardiac
complications, COHb levels of >40% and pregnant women
2-Low protein C levels are not seen with:
Options:
A. Warfarin therapy
B. Neonatal purpura fulminans
C. Factor V Leiden
D. Chicken pox
E. Meningococcal septicaemia
C
Teaching Notes for Question 2
Theme: Low protein C levels
Protein C is a vitamin K-dependent protein synthesised in the liver. It circulates in an inactive form,
which becomes activated by thrombin/thrombomodulin complexes on the surface of endothelial cells.
Activated protein C inhibits coagulation by degrading coagulation factors Va and VIIIa and resulting in
decreased thrombin formation. It requires protein S as a cofactor.
* homozygous deficiency - those with severe deficiency - (protein C levels <1% of normal) present
with severe neonatal purpura fulminans, cerebral thrombosis and DIC. Those with milder defects have
protein C levels of 10 -24 % of normal. They can present with massive venous thrombosis as older
children.
* heterozygous deficiency (protein C levels 30-40% of normal) most frequently presents after puberty
with deep venous thrombosis of the lower limb. It can also present with recurrent superficial
thrombophlebitis and DVT. Most thrombotic episodes occur spontaneously but known associated risk
factors include surgery with immobilisation, pregnancy and the oral contraceptive.
Many people remain asymptomatic. A positive family history of thrombosis is associated with an
increased risk of symptoms - + - 50% of individuals with heterozygous protein C deficiency and a
family history of thrombosis, will experience a thrombotic event.
155
PEDIATRICS
EXAMINATION REVIEW
3-Choose the correct matching pair of disease and antibody out of the following options:Options:
A. Anti-liver/kidney microsomal antibodies (anti LKM-1) in type I autoimmune hepatitis
B. Anti-smooth muscle antibodies in Primary biliary cirrhosis
C. Anti-nuclear antibodies in Primary biliary cirrhosis
D. Anti-mitochondrial antibodies in type II autoimmune hepatitis
E. Anti liver cytosol antibodies in primary sclerosing cholangitis
C
Teaching Notes for Question 3
Theme: Antibodies in GI diseases
Autoimmune hepatitis type I - Anti nuclear antibodies, Anti smooth muscle antibodies, Anti
mitochondrial antibodies
type II - Anti liver/kidney microsomal antibodies (LKM-1)
Anti liver cytosol
Primary Biliary Cirrhosis - Anti nuclear antibodies, Anti mitochondrial (M2) antibodies
Primary Sclerosing cholangitis - ANCA positive .
4-hich one of the following cells in the lung parenchyma produces surfactant?
Options:
A. Alveolar macrophage
B. Endothelial cell
C. Goblet cell
D. Type I pneumocyte
E. Type II pneumocyte
E
Teaching Notes for Question 4
Theme: Surfactant production
Type II pneumocytes produces surfactant in the lungs. Type I pneumocytes, which are as
many as type II pneumocytes, cover 95% of the epithelial surface area. Its key function is
gaseous exchange.
5-Which of the following conditions with renal involvement is the most likely to
have normal complement levels:Options:
A. Grade IV lupus nephritis
B. Post-streptococcal glomerulonephritis
C. Subacute bacterial endocarditis
D. Type II cryoglobulinaemia
E. IgA nephropathy (Buergers disease)
E
Teaching Notes for Question 5
Theme: Renal disease and complement levels
Primary membranous nephropathy and Buergers disease can have normal
complement levels. The other conditions are all associated with activation of the
.classical complement pathway and immune complex formation
156
PEDIATRICS
EXAMINATION REVIEW
A. Lung transplantation
B. Interferon-1-Beta
C. Azathioprine
D. Cyclophophamide
E. High -dose steroids
157
PEDIATRICS
EXAMINATION REVIEW
8-What is the diagnosis that can you infer from the X-ray abdomen?
Options:
A. LBO (Large bowel obstruction)
B. SBO (Small bowel obstruction)
C. Toxic Megacolon
D. Chagas Disease
E. Ankylosing Spondylitis
158
PEDIATRICS
EXAMINATION REVIEW
formation of microvilli and phagocytic vacuoles- structures which are affected by actin
reorganisation. There is also reduced expression of CD43 (sialophorin) on the surface of Band T-lymphocytes.
Patients present with eczema, thrombocytopaenia and recurrent bacterial infections. They
also have an increased risk of lymphoid malignancy. Initially lymphocyte numbers are
normal, and the major defect is impaired production of antibody to thymus independent
polysaccharide antigen. Later lymphocyte numbers decrease, and immunodeficiency
becomes more severe. Patients have recurrent respiratory infections and are susceptible to
organisms such as Pneumocystis carinii and herpes simplex virus. There is impaired cell
mediated immunity (with lymphopenia) and a characteristic pattern of immunoglobulin
levels:- low IgM, normal levels of IgG and high levels of IgA and IgE. Platelet numbers are
decreased and platelets are smaller than usual. These children can present with lifethreatening bleeding. Bone marrow transplant is curative.
Ataxia telangiectasia is caused by defective DNA repair mechanisms. Patients present with
progressive neurological deterioration, ataxia, malignant disease, impaired cell mediated
immunity and defective antibody production.
Adenosine deaminase defiency causes about 20% of cases of SCID.
10-All of the following are type II hypersensitivities except:Options:
A. Production of autoantibodies to acetylcholine receptors in myasthenia gravis
B. Haemolysis following transfusion of blood bearing mismatched blood group antigens
C. The Arthus reaction
D. Hyperacute graft rejection
E. Quinidine-induced thrombocytopenia
C
Teaching Notes for Question 10
Theme: Type II hypersensitivities.
Myasthenia gravis, autoimmune haemolysis, drug-induced thrombocytopenia and
hyperacute graft rejection are all examples of type II hypersensitivity reactions.
Type II hypersensitivity reactions involve antibody-mediated destruction of cells. Antibody usually IgG or IgM can destroy cells by activating complement, or promoting phagocytosis
of antibody-coated cells. They can also bind to cell surface receptors interfering with
normal function.
An example of antibody interfering with normal receptor function occurs in myasthenia
gravis. In this disease, autoantibodies bind to acetylcholine receptors at the neuromuscular
junction. This interferes with normal motor end plate function and also results in a
reduction in receptor numbers.
Blood transfusion reactions exemplify type II hypersensitivity. Host antibodies (usually
IgM) react with foreign antigens on the incompatible transfused blood cells and lead to
complement-mediated haemolysis of these cells.
The Arthus reaction is a type III hypersensitivity reaction mediated by the local deposition
of immune complexes with consequent complement activation and neutrophil infiltration.
Hyperacute rejection is usually due to preformed anti-donor antibody.
These antibodies are directed against foreign MHC molecules of other alloantigens
11-Polycystic kidney disease is:
Options
A. Usually fatal at term
B. Characterised histologically by tubule dysplasia and cystic tissue throughout the renal parenchyma
C. Always clinically apparent at birth
D. Characterised histologically by micro- and macroscopic cysts distributed throughout the renal
parenchyma
E. Kown to exist in 4 genetic forms
159
PEDIATRICS
EXAMINATION REVIEW
160
PEDIATRICS
EXAMINATION REVIEW
161
PEDIATRICS
EXAMINATION REVIEW
162
PEDIATRICS
EXAMINATION REVIEW
163
PEDIATRICS
EXAMINATION REVIEW
164
PEDIATRICS
EXAMINATION REVIEW
24-What does this ECG of an 8-year-old boy with Ebsteins anomaly show?
Options:
A. Atrial flutter
B. Supraventricular tachycardia
C. Sinus tachycardia
D. AV reentrant tachycardia due to accessory pathway
E. Ectopic atrial tachycardia
165
PEDIATRICS
EXAMINATION REVIEW
166
PEDIATRICS
EXAMINATION REVIEW
167
PEDIATRICS
EXAMINATION REVIEW
168
PEDIATRICS
EXAMINATION REVIEW
169
PEDIATRICS
EXAMINATION REVIEW
170
PEDIATRICS
EXAMINATION REVIEW
Interphase FISH on uncultured cells is currently used in rapid prenatal diagnosis. FISH
analysis is used as a second line of investigation after conventional light microscopy. One
of the main research uses of FISH is to localise genes on chromosomes.
171
PEDIATRICS
EXAMINATION REVIEW
172
PEDIATRICS
EXAMINATION REVIEW
173
PEDIATRICS
EXAMINATION REVIEW
174
PEDIATRICS
EXAMINATION REVIEW
175
PEDIATRICS
EXAMINATION REVIEW
176
PEDIATRICS
EXAMINATION REVIEW
G6PDH catalyses the synthesis of NADPH from the hexose monophosphate pathway. The
NADPH is then used to reduce glutathione and hence in conjunction with glutathione
reductase reduces cellular oxidative damage.
G6PDH enzyme deficiency, caused by mutations in the X-linked (Xq28) G6PD gene, affects
over half a billion people worldwide. It is a balanced polymorphism associated with
resistance to falciparum malaria in heterozygous females. This evolutionary advantage
outweighs the small negative effect of affected hemizygous males. At least 400 G6PD
variants have been identified. The various mutations lead to altered expression of G6PD or
an altered half-life of the enzyme.
There are a number of disease manifestations:
- patients may be asymptomatic
- patients may present with neonatal jaundice
- usually the deficiency manifests as acute haemolytic crises provoked by oxidising stress
e.g. certain foods - fava bean, chemicals - naphthalene, certain drugs - dapsone,
primaquine, chloroquine, aspirin (high doses), sulphonamides, nitrofurantoin, vitamin K
and nalidixic acid.
Infections can also provoke haemolysis. Haemolytic crises vary in severity, rarely chronic
haemolysis may occur. If severe, G6PD deficiency can cause immunodeficiency by reducing
the NADPH required for activation of the NADPH oxidase enzyme (cf. CGD).
47-Which of the following statements correctly describes the clinical phenotypes which are
associated with a deficiency of the various complement components?
Options:
A. C5-C9 deficiency and mycoplasma infection
B. C1q deficiency and angio-oedema
C. C2 deficiency and rheumatoid arthritis
D. C3 def is associated with partial lipodystrophy
E. A raised C4 and low C3 in SLE is associated with active disease
177
PEDIATRICS
EXAMINATION REVIEW
178
PEDIATRICS
EXAMINATION REVIEW
likely to be?
Options
A. Oculomotor nerve
B. Trigeminal nerve
C. Facial nerve
D. Vagus nerve
E. Cervical sympathetic trunk
179
PEDIATRICS
EXAMINATION REVIEW
180
PEDIATRICS
EXAMINATION REVIEW
* Adenosine does not terminate atrial flutter, but does increase the degree of AV block
this slows the ventricular rate transiently and may reveal flutter waves on ECG, which can
be diagnostic.
* The circuit of current usually occurs via an anatomically distinct pathway, which can be
interrupted by radiofrequency ablation.
* Most atrial flutter originates in the RA, with a counterclockwise circuit around the
tricuspid valve annulus.
56-The following statements are true about Tetralogy of Fallot (TOF) except:
Options:
A. Children with TOF are susceptible to cerebral abscess
B. The chest X-ray shows oligaemic lung fields
C. The second heart sound is soft and single
D. TOF is associated with dextroposition of the aorta
E. This disorder results in a left to right shunt
181
PEDIATRICS
EXAMINATION REVIEW
B. A renal consultation is advised if the serum creatinine rises above 250 mol/l.
C. All patients with confirmed microalbuminuria, if no contraindications, should be on an ACE inhibitor.
D. Blood pressure is less important than reducing the degree of proteinuria.
E. Calculating ratios of albumin and creatinine in the urine is not as useful as the total urine protein
level.
182
PEDIATRICS
EXAMINATION REVIEW
60-Following a dislocated shoulder in a rugby match that gets out of hand, a patients
axillary nerve is damaged. What is the likely result?
Options:
A. The teres major atrophies.
B. The rounded contour of the shoulder disappears.
C. A loss of sensation may occur in the lateral forearm.
D. The patient may lose the ability to adduct the arm.
E. The patient may have a wrist drop.
183
PEDIATRICS
EXAMINATION REVIEW
9.9 g/dl
83 fl
3.1 x 109/l
117 x 109/l
2 g/l
71 mm in the first hour
134 mmol/l
5.1 mmol/l
10.1 mmol/l
178 mmol/l
30 g/l
80 g/l
Normal
7.8 mmol
Clotting screen:
PTT
APTT
VDRL
11 seconds
39 seconds
Positive 1:320
184
PEDIATRICS
EXAMINATION REVIEW
Negative
Not detected
64-A boy presents with fever, arthralgia and headache and a red, annular rash in his axilla,
which his mother says has been present for one week and has increased in size. Which is
the most likely diagnosis?
Options:
A. Kawasaki disease
B. Infectious mononucleosis
C. Scarlet fever
185
PEDIATRICS
EXAMINATION REVIEW
D. Lyme disease
E. Rheumatic fever
186
PEDIATRICS
EXAMINATION REVIEW
Malaria causes jaundice because of red cell haemolysis and should always be considered in
a child with fever who has been abroad in the past 3 months.
Toxocariasis (organism: the nematode Toxocara canis from dogs, or Toxocara catis from
cats) presents in one of 2 forms:
* Visceral larva migrans is a syndrome of fever, hepatomegaly, wheeze and eosinophilia
caused by the immune response to migrating larvae.
* Ocular larva migrans causes a granulomatous reaction in the retina where the larva dies
and may be detected on fundoscopy in a child being investigated for strabismus or
decreased visual acuity.
Lyme disease (organism: Borrelia burgdorferi) usually presents with skin lesions (erythema
chronicum migrans), fever, arthralgia, and headache. Disease can localise to heart, CNS or
joints.
Cat scratch fever (organism: gram negative bacillus Bartonella henselae) presents with
(usually cervical) lymphadenopathy and a history of a cat scratch.
67-A two-month-old child had his first DTP injection 3 days previously and his mother has
returned because his thigh is swollen. On examination, most of the anterolateral surface of
the thigh is red and indurated. What will you recommend for this child in the future?
Options
A. He should not have any further immunisations
B. He should have all immunisations except pertussis
C. He should receive the acellular pertussis vaccine when next immunised
D. He should receive the oral pertussis vaccine when next immunised
E. He should receive the whole cell pertussis vaccine (as DTP) as usual at his next immunisation
187
PEDIATRICS
EXAMINATION REVIEW
188
PEDIATRICS
EXAMINATION REVIEW
can also result in acidosis with high anion gap. Drugs causing normal anion gap acidosis
include amphotericin, acetazolamide, and solvents (toluene) *glue*. Another cause of
normal anion gap acidosis is renal tubular acidosis.
Glue poisoning usually gives rise to a distal (type 1) RTA, therefore a hyperchloremic
acidosis with a normal anion gap.
71-The most important cause of death in infectious mononucleosis is:
Options:
A. Liver failure.
B. Kidney failure.
C. Heart failure.
D. Respiratory failure.
E. Splenic rupture.
189
PEDIATRICS
EXAMINATION REVIEW
9.9 g/dl
83 fl
3.1 X 109/l
117 X 109/l
2 g/l
71 mm in the first hour
134 mmol/l
5.1 mmol/l
10.1 mmol/l
178 mmol/l
30 g/l
80 g/l
normal
7.8 mmol
11 seconds
39 seconds
positive 1:320
negative
not detected
190
PEDIATRICS
EXAMINATION REVIEW
The patient then develops acute dyspnoea and examination reveals decreased air entry at the right
base, tachycardia, BP 105/55. O2 sats 94% on room air.
She responds well to prednisolone and azathioprine, but at an OPD appointment 3 months later is
complaining of worsening right hip pain poorly responsive to NSAIDs.
X-ray is normal but bone scan shows increased uptake in the hip.
What is the most likely cause of her hip pain?
Options:
A. Osteoporosis
B. Avascular necrosis
C. Septic arthritis
D. Stress fracture
E. Trochanteric bursitis
191
PEDIATRICS
EXAMINATION REVIEW
Examination reveals no focal signs of infection. Blood cultures are taken and a CXR is
unremarkable. She is treated with cefotaxime, however after 48 hours the fevers continue
and a repeat CXR shows left lower lobe consolidation.
Options:
A. Streptococcus pneumoniae
B. Klebsiella pneumoniae
C. Pneumocystis carinii
D. Cytomegalovirus
E. Cryptococcus neoformans
192
PEDIATRICS
EXAMINATION REVIEW
193
PEDIATRICS
EXAMINATION REVIEW
194
PEDIATRICS
EXAMINATION REVIEW
The nuchal scan together with bHCG and PAP identifies those at high risk of a Downs
syndrome pregnancy in >80% of cases.
84-A 17 year old student develops a deterioration in vision in her left eye over three days.
She complains of discomfort in the eye and thinks that difficulty with perception of colour
was the first problem that she noticed, during a trip to an art gallery. On examination, visual
acuity on the left is down to light perception. The pupil appears dilated and does not
constrict to light, although does when a torch is shone in the right eye.
What is the most likely diagnosis :Options:
A. CMV retinitis
B. Optic neuritis
C. Anterior iscaemic optic neuropathy
D. Amaurosis Fugax
E. Central retinal artery occlusion
195
PEDIATRICS
EXAMINATION REVIEW
196
PEDIATRICS
EXAMINATION REVIEW
197
PEDIATRICS
EXAMINATION REVIEW
198
PEDIATRICS
EXAMINATION REVIEW
199
PEDIATRICS
EXAMINATION REVIEW
B. It is a notifiable disease
C. The patient may be jaundiced during the immune phase
D. The organism may be isolated from urine during the septic phase
E. Ciprofloxacin is the treatment of choice for severe forms
200
PEDIATRICS
EXAMINATION REVIEW
-Cystinosis.
-Galactosaemia.
-Tyrosinaemia.
-Hereditary fructosaemia.
-Lows syndrome (Oculo-Cerebral- Renal syndrome).
-Chemotherapy- esp 1 fosphamide.
-Wilsons disease.
-Heavy metal poisoning.
-Drugs- esp amphetericin.
-Idiopathic- large group.
Cystinosis.
-Autosomal recessive.
-Lysosomal storage disorder- unable to transport cystine out of lysosome.
-Diagnosis clinical suspicion, slit lamp, Wbc cystine level.
-Antenatal diagnosis available.
-All tissues in body involved.
*Kidney-proximal renal tubular acidosis.
*Eyes- corneal deposits.
*Liver- impaired function.
*Pancreas- diabetes.
-Treatment , Phosphocyteamic. Increase cystine-transport out of lysosome, can delay
onset of renal failure.
201
PEDIATRICS
EXAMINATION REVIEW
96-The mother of a baby in the postnatal ward has an antenatal screening test for sickle
haemoglobin. This is positive deposite a normal Hb & MCV. The Hb-electrophoresis shows
HbA 55% and Hbs 45%. Her partner has a microcytic anaemia, and Hb-electrophoresis
shows an HbA band only with HbA2 quantitation of 4.8%.
Which diagnosis is impossible in the baby?
Options:
A. Sickle cell trait.
B. Haematologically normal.
C. B-thalassaemia trait.
D. Sickle-B-thalassaemia.
E. Sickle cell diseas(HbSS).
202
PEDIATRICS
EXAMINATION REVIEW
98-A 18 years old student becomes un well with a sore throat, low grade fever and has some
cervical lymphadenopathy,
on consulting his GP 3 days after the onset of this illness, examination is otherwise
unremarkable, investigations reveals:
FBC
normal ,Na=137
K=4.4 urea=10.1 ,creatinin =141 ,C3= 1.31g/l (0.551.20)C4= 0.42g/l (0.20-0.50),CH50= 87% (50-110) Urinalysis: blood 3+, protein +, few red
cell casts.
What is the most likely diagnosis?
Options:
A. post-streptococcal glomerulonephritis.
B. Wegeners granulomatosis.
C. Ig A nephropathy.
D. Interstitial nephrosis.
E. Henoch-Schonlein purpura.
99-An adolescent male is admitted with progressive limb weakness, on examination he has
flacid weakness predominantly in the legs and absent reflexes. CSF examination shows
markedly raised protein & a mild lymphocytosis,
Which of the following does not imply a worse prognosis?
Options:
A. History of diarrhea in the preceding few weeks.
B. presence of postural hypotension.
C. Markedly reduced FVC.
D. Evidence demyelination on nerve conduction studies.
E. Sever weakness.
203
PEDIATRICS
EXAMINATION REVIEW
Questions
(PART 3)
PEDIATRICS
EXAMINATION REVIEW
1-Options :
A. Osteomalacia
B. Ankylosing spondylitis
C. Avascular necrosis
D. Rheumatoid arthritis
E. Osteoarthritis
F. Pagets disease
Instructions: Choose the most appropriate disease for the radiological description.
1. Narrowing of joint space, marginal osteophytes and subchondral cysts.
2. Bamboo spine
3. Mixed osteoblastic and osteolytic lesions in the left tibia. There is also cortical
thickening.
4. Joint space narrowing, juxtaarticular osteoporosis and bony erosions.
2-Options :
A. Streptococcal throat infection
B. Haemophilus influenza epiglottitis
C. Pneumococcal infection
D. Kawasakis disease
E. Typhus
F. Tuberculosis
G. Infectious mononucleosis
H. Erythema infectiosum
I. Mumps
J. Varicella zoster
205
PEDIATRICS
EXAMINATION REVIEW
K. Rubella
L. Measles
Instructions: For each of the patients described below choose the single most likely option from the
above list. Each option may be used once, more than once or not at all.
1. A 5 year old girl presents with conjunctivitis, fever and coryza 12 days after exposure to a sick
sibling. On examination she has blue white punctate lesions on her buccal mucosa.
2. A 16 year old boy presents with fever, sore throat, anorexia, malaise. He has enlarged cervical
lymph nodes. On investigation there is lymphocytosis with atypical lymphocytes.
3. A 6 year old girl presents with fever and a slapped cheek erythematous eruption on her
cheeks.
4. A 8 year old boy presents with malaise, fever, and has painful swelling of both parotid glands.
5. A 6 year old boy presents with fever, cervical lymphadenopathy and erythema of palms and
soles.
3-Options :
A. Full blood count
B. Marrow biopsy
C. EEG
D. CT scan
E. INR
F. Thyroid function tests
G. Liver function tests
H. Liver biopsy
I. Prolactine levels
J. Drug serum levels
K. Breast ultrasound
Instructions: Drug side effects-investigations; give the most appropriate investigation for each
scenario. Each answer may be used once, more than once or not at all.
1. Patient on phenytoin, with diplopia, tremor, and low mood
2. Patient on Sodium Valproate, with vomiting, anorexia, jaundice, drowsiness
3. Patient on carbamazepine feeling unwell, with sore troat, and temp. 37.2 C
4. Male patient on phenothiazines comes with discharge from his breast
4-Options
A. Down's syndrome
B. Turner's syndrome
C. Pataus syndrome
D. Cri-du-chat syndrome
E. Cystic fibrosis
F. Kleinfelter's syndrome
G. Kallmanns syndrome
H. Silver Russell syndrome
Instructions: From the given list of options, choose the single best answer for the given set of
questions. Each option maybe used once, more than once or not used at all.
1. A young mother brings her child to the paediatric OPD as she noticed he has absence of skin
on his scalp area. O/E- micrognathia, microcephaly, umbilical hernia and a pansystolic murmur of
VSD are present.
206
PEDIATRICS
EXAMINATION REVIEW
2. A 5-year-old child is brought to the hospital with complaints of repeated chest infections and
GI problems since birth. His sweat chloride levels are diagnostic.
3. A tall adult male, with absence of secondary sexual characters, anosmia, cleft lip and palate
and mostly normal mental development.
4. A young female, with delayed puberty, cubitus valgus, heart defects and mild mental
retardation.
5. A mother brings her 2 year old with respiratory infection to the OPD. O/E he has simian
crease, retarded physically and mentally, large tongue, and a VSD.
5-Options
A. Duodenal atresia
B. Jejunal atresia
C. Oesophageal atresia
D. Necrotising enterocolitis
E. Malrotation
F. Malrotation with volvulus
G. Hirschprung's disease
H. Intussusception
Instructions: Please match the most likely diagnosis with the scenario described below
1. A baby girl born at 26 weeks is transferred to NICU on day 3 with abdominal distention, blood
in the stools and increasing ventilatory requirements
2. A baby girl born at 36 weeks is transferred to NICU on day 1 with cyanosis and respiratory
distress after attempted feeding
3. A baby boy born at term is transferred to NICU on day 2 with abdominal distension and failure
to pass meconium.
4. A baby boy born at term is transferred to NICU on day 10 with sudden collapse. He has bilious
aspirates, some blood in the stool and a silent abdomen.
5. A baby boy born at 32 weeks is transferred to NICU on day 1 with bilious vomitting. On
examination, he has features of Trisomy 21.
6-Options
A. Atrial septal defect
B. Ventricular septal defect
C. Patent ductus arteriosus
D. Coarctation of aorta
E. Ebstein anomaly
Instructions: Match the correct physical sign/presenting symptoms with the lesion of congenital heart
disease:1. A 17 year old presents with headache, leg fatigue and claudication. On examination there is
an absence of femoral pulses with a mid-systolic murmur heard over the back.
2. A 18 year old presents for a health checkup with no symptoms. On examination there is a
widely split fixed 2nd heart sound.
3. An 18 year old presents with palpitations. On auscultation there is a quintuple cardiac
cadence.
4. A 7 year old presents with breathlessness. On auscultation there is a continuous murmur.
5. A 3 year old presents with breathlessness and pedal oedema. On auscultation there is a sharp
holosystolic murmur heard along the left sternal border.
7-An 18-year-old man was admitted to casualty at 3 am having collapsed at a nightclub. A
friend, who had accompanied him in the ambulance but subsequently left A and E before
speaking to a doctor, told the paramedics that his friend had been well earlier in the evening
207
PEDIATRICS
EXAMINATION REVIEW
and that he usually did not drink much alcohol or take any drugs.
Investigations revealed:
Haemoglobin
8.6 x 109/L
White cell count
25.8 x 109/L
Neutrophils
22.5 x 109/L
Lymphocytes
2.0 x 109/L
Monocytes
0.8 x 109/L
Eosinophils
0.4 x 109/L
Basophils
0.1 x 109/L
Platelets
32 x 109/L
Reticulocyte count
12%
Prothrombin time
32 s
Activated partial thromboplastin time 90 s
Fibrinogen
0.3 g/L
D-Dimer screen
3.0 mg/L
Serum sodium
140 mmol/L
Serum potassium
6.3 mmol/L
Serum urea
17.5 mmol/L
Serum creatinine
200 mmol/L
Lactate dehydrogenase
540 IU/L
The blood film showed red cell fragmentation with polychromasia, toxic granulation of
neutrophils and platelet anisocytosis.
What is the diagnosis?
Options :
Choose 1
A. Autoimmune haemolysis
B. Autoimmune thrombocytopaenia
C. Disseminated intravascular coagulation
D. Drug-induced haemolysis
E. Thrombotic thrombocytopenic purpura
8-Options
A. Intussusception
B. Mesenteric adenitis
C. Meckel's diverticulum
D. Appendicitis
E. Strangulated inguinal hernia
F. Hypertrophic pyloric stenosis
G. Constipation
H. Nephroblastoma
I. Riedel's lobe of liver
Instructions: Choose the single most likely diagnosis from the options listed above for each of the
clinical scenarios described below. Each option may be used once, more than once or not at all.
1. A 4 year old boy is brought into hospital by his mother. For the past 2 weeks he has had a
cold, a sore throat, headache and nausea. Today he has vomited twice, is not tolerating fluids,
and complains of central abdominal pain.
2. A 2 year old girl is referred urgently to clinic having presented to her general practitioner with
haematuria on two occassions, and an abdominal mass. She is now asymptomatic. There is a
firm mass with a poorly defined smooth lower border in the left upper quadrant of her abdomen.
On percussion of the liver there is no dullness, and on palpation the liver is non-tender.
3. A 13 year old girl presents with a complaint of abdominal pain that she has experienced for
208
PEDIATRICS
EXAMINATION REVIEW
the past two weeks, which has worsened acutely today. History reveals a bright red loss of blood
per rectum with defaecation, which she notes on the toilet paper. Her abdomen is soft on
palpation and on auscultation there are normal bowel sounds. A per rectum examination is too
tender to perform and an anal fissure is noted at 6 o'clock.
4. A 1 year old child has been noted to have bouts of screaming where he is doubled-up in agony
from abdominal pain with a 60 minute pain-free period in between. When you ask to see his
nappy, you note a redcurrant jelly stool, with a trace of blood on his anus.
9-A 18-years-old adolescent had a bone marrow transplant from his sister for treatment of
acute myeloid leukemia. Six weeks later he presented with a rash, general malaise, poor
appetite and chronic watery, green diarrhoea. His medications included cyclosporin,
penicillin V, co-trimoxazole and aciclovir.
On examination, a diffuse macular rash was visible, mainly affecting the palms of his hands
and the soles of his feet. His pulse was 104 beats per minute (bpm) regular. Blood pressure
112/74 mmHg. Heart sounds were unremarkable and his chest was clear. His abdomen was
soft apart from mild tenderness in the right upper quadrant. No organs were palpable. Bowel
sounds were active. Neurological examination was normal.
Investigations revealed:
Haemoglobin
12.4 g/dL
Mean corpuscular volume (MCV)
90 fL
White cell count
3.65 x 109/L
Neutrophils
1.7 x 109/L
Lymphocytes
1.0 x 109/L
Monocytes
0.8 x 109/L
Eosinophils
0.05 x 109/L
Basophils
0.1 x 109/L
Platelets
147 x 109/L
Serum albumin
36 g/L
Serum total bilirubin
64 mol/L
Serum alanine transferase
90 IU/L
Serum alkaline phosphatase
135 IU/L
Serum gamma glutamyl transferase
40 IU/L
What is the diagnosis?
Options:
Choose 1
A. Acute graft versus host disease
B. Clostridium difficile infection
C. Cytomegalovirus infection
D. Side-effect of immunosuppression
E. Transfusion-associated graft versus host disease
10-A 14 year old girl was referred to the local child psychiatrist by her GP who was
concerned about her weight loss. The GP reported that she has lost about 2 stones over the
past 2 months and that although she was preoccupied by preparing food, she did not seem to
be eating well. Her mother reported that she has been hording food in her bedroom. The
mother also informed the GP that despite losing weight her daughter was spending long time
every day undertaking heavy exercises. Which of the following supports a diagnosis of
anorexia nervosa in this patient:Options:
Choose 5
209
PEDIATRICS
EXAMINATION REVIEW
B. Bone scan
C. MRI
D. Anti dsDNA and complement
E. Thomas test
F. Pelvic spring test
G. Sigmoidoscopy, biopsy and enema
H. Joint aspiration and microscopy
I. Clinical history alone, joint X-ray
J. Joint aspiration, swab and blood cultures
K. Rheumatoid factor, joint X-ray
L. ECHO
M. Neurophysiology
Instructions: For each of the patients described below choose the single most likely option from the
above list of options. Each option may be used once, more than once or not at all.
1. A 5-year-old Asian refugee child develops high fevers, organomegaly, joint pains and a rash.
He also has a 4/6 murmur.
2. A 45-year-old man who was on a fast developed acute pain in his toe.
3. A 45-year-old woman presents with wrist pain, which is worse at night and relieved by
dangling the hand over the edge of the bed and shaking it.
4. A 45-year-old man presents with swollen, painful and stiff hands and feet and the symptoms
are worse in the mornings.
5. A man has a bike accident and injures his leg. There is a small open wound. The wound is
treated but 2 days later he develops pain in his knee.
12-Options
A. Varicella
B. Scabies
C. Pityriasis rosea
D. Psoriasis
E. Meningitis
F. Measles
G. Kawasakis disease
H. Herpes simplex virus
I. Ebstein-Barr virus
J. Roseola infantum
K. Impetigo
L. Erythema toxicum
Instructions: For each of the patients described below choose the single most likely option from the
above list of options: Each option may be used once, more than once or not at all.
1. A 6-month-old boy presents with a generalised rash after a bout of fever.
2. A 3-year-old boy presents with oedema, congested conjunctivae, erythema of the oral cavity,
lips, and palms. It does not respond to antibiotics.
3. A mother rings A&E saying that her 3-year-old boy has fever for 1 day and is drowsy. He also
has a rash which does not blanch on pressure.
4. A 17-year-old African male presents with sore throat, fever, anorexia, malaise,
lymphadenopathy, palatal petechiae, and splenomegaly.
5. A non-immunised child of 5 years presents with catarrh, conjunctivitis, fever. He has some
bluish white spots on his buccal mucosa.
210
PEDIATRICS
EXAMINATION REVIEW
13-Options :
A. Deformation
B. Disruption
C. Malformation
D. Dysplasia
E. Teratogen
F. Dystocia
Instructions: Match the above options with the best descriptive statement from the choices given
below:1. Chemical agent with a malign influence on development
2. Abnormal development of a structure caused by extrinsic forces
3. Intrinsic defects in the pattern of development
4. Destruction of normally formed tissue
5. Abnormal organisation of cells in tissue
14-Options :
A. Herpes virus
B. Streptococcus
C. Staphylococcus
D. Brucellosis
E. Tuberculosis
F. Poliomyelitis
G. Candidiasis
H. Infectious mononucleosis
I. Rubella
J. Diphtheria
K. Coryza
L. Meningitis
M. Hepatitis
N. Influenza
Instructions: For each of the patients described below choose the single most likely option from the
above list of options: (Each option may be used once, more than once or not at all.)
1. A 4-year-old boy presents to A&E with shock. He is found to have myocarditis. His mother
gives a history of tonsillitis.
2. A 15-year-old patient presents with a rash and suboccipital lymphadenopathy. He was not
immunised as a child.
3. A 35-year-old lady presents with myalgia, nausea, and fever. She also has abdominal pain.
Her bilirubin is raised.
4. A veterinarian presents with fever, malaise, anorexia, vomiting, hepatosplenomegaly,
depression, arthritis, sacro-ileitis and rash.
5. A man who is on steroids for rheumatoid arthritis for a long period of time now presents with
white patches in his mouth.
211
PEDIATRICS
EXAMINATION REVIEW
212
PEDIATRICS
EXAMINATION REVIEW
1-Options :
A. Osteomalacia
B. Ankylosing spondylitis
C. Avascular necrosis
D. Rheumatoid arthritis
E. Osteoarthritis
F. Pagets disease
Instructions: Choose the most appropriate disease for the radiological description.
1. Narrowing of joint space, marginal osteophytes and subchondral cysts.
2. Bamboo spine
3. Mixed osteoblastic and osteolytic lesions in the left tibia. There is also cortical
thickening.
4. Joint space narrowing, juxtaarticular osteoporosis and bony erosions.
Teaching Notes for Question 1
Theme: Radiographic abnormalities in rheumatological disease
Rheumatoid arthritis and osteoarthritis features are as what given in the question.
Ankylosing spondylitis affects the spine in young adults and causes restricted spinal mobility. In late
stages X-ray shows bamboo spine where vertebrae are fused.
Pagets disease is due to a disorder of bone remodelling. It is usually monoostotic (affects one bone).
Most important biochemical abnormality is elevated alkaline phosphatase and normal calcium levels.
2-Options :
A. Streptococcal throat infection
B. Haemophilus influenza epiglottitis
C. Pneumococcal infection
213
PEDIATRICS
EXAMINATION REVIEW
D. Kawasakis disease
E. Typhus
F. Tuberculosis
G. Infectious mononucleosis
H. Erythema infectiosum
I. Mumps
J. Varicella zoster
K. Rubella
L. Measles
Instructions: For each of the patients described below choose the single most likely option from the
above list. Each option may be used once, more than once or not at all.
1. A 5 year old girl presents with conjunctivitis, fever and coryza 12 days after exposure to a
sick sibling. On examination she has blue white punctate lesions on her buccal mucosa.
2. A 16 year old boy presents with fever, sore throat, anorexia, malaise. He has enlarged
cervical lymph nodes. On investigation there is lymphocytosis with atypical lymphocytes.
3. A 6 year old girl presents with fever and a slapped cheek erythematous eruption on her
cheeks.
4. A 8 year old boy presents with malaise, fever, and has painful swelling of both parotid
glands.
5. A 6 year old boy presents with fever, cervical lymphadenopathy and erythema of palms and
soles.
Teaching Notes for Question 2
Theme: Diagnosis of childhood illness.
Q1. This is a case of measles caused by an RNA paramyxovirus spread by droplets. Incubation period
is 8 to 14 days. Measles infection presents with catarrh, conjunctivitis and fever. Koplik spots are
pathognomonic 'grain of salt like' spots on the buccal mucosa located adjacent to the posterior molar.
They are seen early in the illness. A maculopapular rash typically first appears behind the ears and
spreads to involve the face and then the trunk and limbs. Complications are due to infection by the
measles virus itself
-respiratory- laryngotracheobronchitis, bronchitis, giant celll pneumonia
- ear - otitis media
-CNS - postinfectious encephalomyelitis. This severe complication usually manifests 7-14 days after
initial presentation with drowsiness, seizures, focal neurological signs and coma. Mortality is 10% and
15% of patients who recover will have residual neurological deficit.
- subacute sclerosing panencephalitis (SSPE) is a rare complication caused by latent measles virus
infection which on reactivation 5-10 yrs later results in a degenerative CNS disorder.
Secondary bacterial infection is an important complication of measles infection particularly in
malnourished children.
Those wishing to update themselves on the current debate about measles vaccine and autism should
read the section and refer to the journals about measles vacccine at web address
http://www.doh.gov.uk/mmr/index.html.
Q2. This is a case of infectious mononucleosis caused by Epstein Barr virus - a virus of the herpes
group which infects B-lymphocytes. It presents usually in young adults with fever, sore throat,
anorexia, malaise, lymphadenopathy, palatal petechiae, splenomegaly, hepatitis and haemolytic
anaemia. A mild erythematous maculopapular rash occurs in 3-15% of patients - this number increases
to 80% of patients if there has been previous treatment with amoxicillin or ampicillin. Peripheral blood
count reveals lymphocytosis, and the smear will show atypical lymphocytes. These are CD8+ Tlymphocytes which are produced in response to EBV infected B-cells.
Q3. This is a case of erythema infectiosum also known as slapped cheek syndrome. It is caused by
parvovirus B19 a small single-strand DNA virus. Common symptoms include a mild prodromal illness
consisting of fever, malaise, headache, myalgia, nausea, and rhinorrhea. On day 5-7 after symptom
onset, a bright red rash appears on the cheeks, often followed by an erythematous maculopapular rash
on the trunk and legs. Resolution of the rash reveals a lacy reticular pattern on the limbs.
Complications include
-arthritis- typically asymmetrical and transient
-aplastic crises - seen in children with conditions which decrease red cell survival such as sickle cell
214
PEDIATRICS
EXAMINATION REVIEW
3-Options :
A. Full blood count
B. Marrow biopsy
C. EEG
D. CT scan
E. INR
F. Thyroid function tests
G. Liver function tests
H. Liver biopsy
I. Prolactine levels
J. Drug serum levels
K. Breast ultrasound
Instructions: Drug side effects-investigations; give the most appropriate investigation for each
scenario. Each answer may be used once, more than once or not at all.
1. Patient on phenytoin, with diplopia, tremor, and low mood
2. Patient on Sodium Valproate, with vomiting, anorexia, jaundice, drowsiness
3. Patient on carbamazepine feeling unwell, with sore troat, and temp. 37.2 C
4. Male patient on phenothiazines comes with discharge from his breast
Teaching Notes for Question 3
Theme: Drug(s)
Q1. (J) Phenytoin levels vary in individuals; plasma concentration monitoring is advised on starting the
treatment.
Q2. (G) Liver failure is more common in children < 3 years old , but may occur in any age, LFT should
be monitored in first 6 months of therapy
Q3. (A) Blood dyscrasia with low WBC and frequent infections is a severe side effect.
Q4. (I)Neuroleptics by blocking dopamine D2 receptors may give extrapyramidal effects and
215
PEDIATRICS
EXAMINATION REVIEW
hyperprolactinemia.
4-Options
A. Down's syndrome
B. Turner's syndrome
C. Pataus syndrome
D. Cri-du-chat syndrome
E. Cystic fibrosis
F. Kleinfelter's syndrome
G. Kallmanns syndrome
H. Silver Russell syndrome
Instructions: From the given list of options, choose the single best answer for the given set of
questions. Each option maybe used once, more than once or not used at all.
1. A young mother brings her child to the paediatric OPD as she noticed he has absence of skin
on his scalp area. O/E- micrognathia, microcephaly, umbilical hernia and a pansystolic murmur
of VSD are present.
2. A 5-year-old child is brought to the hospital with complaints of repeated chest infections and
GI problems since birth. His sweat chloride levels are diagnostic.
3. A tall adult male, with absence of secondary sexual characters, anosmia, cleft lip and palate
and mostly normal mental development.
4. A young female, with delayed puberty, cubitus valgus, heart defects and mild mental
retardation.
5. A mother brings her 2 year old with respiratory infection to the OPD. O/E he has simian
crease, retarded physically and mentally, large tongue, and a VSD.
Teaching Notes for Question 4
Theme: Chromosomal disorders
1. Pataus syndrome is trisomy 13.
2. Cystic fibrosis occurs due to abnormality in long arm of chromosome 7.
3. Kallmans syndrome is characterised with the features given in the question.
4. Turners syndrome is 45 XO karyotype.
5. Downs syndrome is trisomy 21.
5-Options
A. Duodenal atresia
B. Jejunal atresia
C. Oesophageal atresia
D. Necrotising enterocolitis
E. Malrotation
F. Malrotation with volvulus
G. Hirschprung's disease
H. Intussusception
Instructions: Please match the most likely diagnosis with the scenario described below
1. A baby girl born at 26 weeks is transferred to NICU on day 3 with abdominal distention,
blood in the stools and increasing ventilatory requirements
2. A baby girl born at 36 weeks is transferred to NICU on day 1 with cyanosis and respiratory
distress after attempted feeding
3. A baby boy born at term is transferred to NICU on day 2 with abdominal distension and
failure to pass meconium.
4. A baby boy born at term is transferred to NICU on day 10 with sudden collapse. He has
bilious aspirates, some blood in the stool and a silent abdomen.
216
PEDIATRICS
EXAMINATION REVIEW
5. A baby boy born at 32 weeks is transferred to NICU on day 1 with bilious vomitting. On
examination, he has features of Trisomy 21.
Teaching Notes for Question 5
Theme: Neonatal intestinal obstruction
1. NEC is a disease of prematurity and impaired gut perfusion is thought to be responsible for the
clinical picture.
2. Oesophageal atresia has an incidence of about 1 in 2500 live births. There is usually an associated
tracheo-oesophageal fistula (90%). There is often polyhydramnios in pregnancy. The child presents
with frothing from the mouth, choking, coughing with episodes of cyanosis exacerbated by feeding.
There is a high incidence of aspiration pneumonia.
3. Hirschprung's disease is caused by the absence of ganglion cells in the myenteric plexuses of the
distal bowel. Aganglionosis may be restricted to the rectum or rectosigmoid region - 'short segment
Hirschsprung's disease', or may extend above the sigmoid - 'long segment Hirschsprung's disease'. In
'short segment Hirschsprung's disease', there is a higher incidence in males (4:1) with an equal sex
incidence in 'long segment Hirschsprung's' disease.
The disorder shows a familial tendency, and there is an association with Down's syndrome (5-15%)
and cardiac malformation (2-5%).
Clinical symptoms are usually present within the first few days of life with delayed passage of
meconium, abdominal distension, bilious vomiting, poor feeding and failure to thrive. Enterocolitis is a
severe complication and presents with diarrhoea, gross abdominal distension and circulatory collapse.
Children with 'short segment Hirschsprung's disease' can present later in life, with a history of chronic
constipation since birth and poor weight gain. Barium enema shows a narrowed distal colon with
proximal dilation. A second enema 24 hrs after the first shows retention of contrast and may reveal the
transition zone between normal bowel and the distal aganglionic intestine. Rectal biopsy confirms the
diagnosis.
Treatment usually involves an initial colostomy proximal to the aganglionic bowel. This is followed by a
definitive pull through procedure at 3-12 months of age.
4. During embryological development the intestines herniate into the umbilical cord at 4- 10 weeks
gestational age. They then return to the abdomen and rotate in an anticlockwise direction.
Failure of this normal process results in malrotation which results in a narrow mesenteric stalk which
predisposes to midgut volvulus and intestinal obstruction. Babies present with vomiting, and in cases
of volvulus with abdominal distension, rectal passage of blood and peritonitis.
5. Duodenal atresia occurs in 1 in 10,000 live births.The site of obstruction is most commonly in 2nd
part of duodenum with hypertrophy of the proximal duodenum. There is an association with Down's
syndrome (30%) as well as cardiac anomalies, malrotation and biliary atresia. Babies present with
bilious or non-bile stained vomiting. X-ray shows the typical 'double-bubble'.Treatment is surgicalduodenoduodenostomy.
Causes of bowel obstruction in the neonate are:
-atresia of any part of the gastrointestinal tract
-duodenal, jejunoileal atresia, large bowel atresia
-mechanical obstruction
-meconium ileus, meconium plug
-volvulus
-malrotation
-imperforate anus
-failure of normal peristalsis - Hirschsprung's disease
-intussusception may occasionally occur in the neonate
Later in the neonatal period other conditions should be considered, including pyloric stenosis.
6-Options
A. Atrial septal defect
B. Ventricular septal defect
C. Patent ductus arteriosus
D. Coarctation of aorta
E. Ebstein anomaly
Instructions: Match the correct physical sign/presenting symptoms with the lesion of congenital heart
disease:1. A 17 year old presents with headache, leg fatigue and claudication. On examination there is
an absence of femoral pulses with a mid-systolic murmur heard over the back.
2. A 18 year old presents for a health checkup with no symptoms. On examination there is a
217
PEDIATRICS
EXAMINATION REVIEW
218
PEDIATRICS
D-Dimer screen
Serum sodium
Serum potassium
Serum urea
Serum creatinine
Lactate dehydrogenase
EXAMINATION REVIEW
3.0 mg/L
140 mmol/L
6.3 mmol/L
17.5 mmol/L
200 mmol/L
540 IU/L
The blood film showed red cell fragmentation with polychromasia, toxic granulation of neutrophils and
platelet anisocytosis.
What is the diagnosis?
Options :
Choose 1
A. Autoimmune haemolysis
B. Autoimmune thrombocytopaenia
C. Disseminated intravascular coagulation
D. Drug-induced haemolysis
E. Thrombotic thrombocytopenic purpura
Teaching Notes for Question 7
Theme: DIC and Ecstasy
The coagulopathy, low fibrinogen, raised D-dimers, anaemia, high neutrophil count and
thrombocytopenia are all indicative of disseminated intravascular coagulation (DIC). In addition the
renal failure with high potassium levels (K) and lactate dehydrogenase (LDH) are suggestive of
associated rhabdomyolysis. The causative agent is likely to be ecstasy/3,4methylenedioxymethamphetamine (MDMA).
8-Options
A. Intussusception
B. Mesenteric adenitis
C. Meckel's diverticulum
D. Appendicitis
E. Strangulated inguinal hernia
F. Hypertrophic pyloric stenosis
G. Constipation
H. Nephroblastoma
I. Riedel's lobe of liver
Instructions: Choose the single most likely diagnosis from the options listed above for each of the
clinical scenarios described below. Each option may be used once, more than once or not at all.
1. A 4 year old boy is brought into hospital by his mother. For the past 2 weeks he has had a
cold, a sore throat, headache and nausea. Today he has vomited twice, is not tolerating fluids,
and complains of central abdominal pain.
2. A 2 year old girl is referred urgently to clinic having presented to her general practitioner with
haematuria on two occassions, and an abdominal mass. She is now asymptomatic. There is a
firm mass with a poorly defined smooth lower border in the left upper quadrant of her abdomen.
On percussion of the liver there is no dullness, and on palpation the liver is non-tender.
3. A 13 year old girl presents with a complaint of abdominal pain that she has experienced for
the past two weeks, which has worsened acutely today. History reveals a bright red loss of blood
per rectum with defaecation, which she notes on the toilet paper. Her abdomen is soft on
palpation and on auscultation there are normal bowel sounds. A per rectum examination is too
tender to perform and an anal fissure is noted at 6 o'clock.
4. A 1 year old child has been noted to have bouts of screaming where he is doubled-up in agony
from abdominal pain with a 60 minute pain-free period in between. When you ask to see his
219
PEDIATRICS
EXAMINATION REVIEW
nappy, you note a redcurrant jelly stool, with a trace of blood on his anus.
Teaching Notes for Question 8
Theme: Abdominal conditions in children
1. Mesenteric adenitis is caused by inflammation of the mesenteric lymph nodes. It is most often
associated with a viral infection, but an association with streptococcal infections of the upper
respiratory tract has been reported. Its clinical presentation mimics that of acute appendicitis. The child
with appendicitis is usually quiet with facial flushing. Enlarged cervical lymph nodes on examination
with a history of a previous URTI points towards mesenteric adenitis.
On examination, the site of abdominal tenderness may shift with a change in patient position, as
opposed to appendicitis, where the location of tenderness tends to be fixed. Pyrexia usually resolves
rapidly with mesenteric adenitis. Unfortunately (for diagnostic purposes) appendicitis has few signs of
peritonitis in the early stages. The longer history of this case suggests mesenteric adenitis as the single
most likely diagnosis. If the child is not admitted for observation, there should be careful discussion
with the parents to observe for worsening symptoms despite analgesics and fluids. This will prevent a
diagnosis of appendicitis being missed.
9-A 18-years-old adolescent had a bone marrow transplant from his sister for treatment of
acute myeloid leukemia. Six weeks later he presented with a rash, general malaise, poor
appetite and chronic watery, green diarrhoea. His medications included cyclosporin,
penicillin V, co-trimoxazole and aciclovir.
On examination, a diffuse macular rash was visible, mainly affecting the palms of his hands
and the soles of his feet. His pulse was 104 beats per minute (bpm) regular. Blood pressure
112/74 mmHg. Heart sounds were unremarkable and his chest was clear. His abdomen was
soft apart from mild tenderness in the right upper quadrant. No organs were palpable. Bowel
sounds were active. Neurological examination was normal.
Investigations revealed:
Haemoglobin
Mean corpuscular volume (MCV)
White cell count
Neutrophils
Lymphocytes
Monocytes
Eosinophils
Basophils
Platelets
Serum albumin
Serum total bilirubin
Serum alanine transferase
Serum alkaline phosphatase
Serum gamma glutamyl transferase
12.4 g/dL
90 fL
3.65 x 109/L
1.7 x 109/L
1.0 x 109/L
0.8 x 109/L
0.05 x 109/L
0.1 x 109/L
147 x 109/L
36 g/L
64 mol/L
90 IU/L
135 IU/L
40 IU/L
220
PEDIATRICS
EXAMINATION REVIEW
10-A 14 year old girl was referred to the local child psychiatrist by her GP who was
concerned about her weight loss. The GP reported that she has lost about 2 stones over the
past 2 months and that although she was preoccupied by preparing food, she did not seem to
be eating well. Her mother reported that she has been hording food in her bedroom. The
mother also informed the GP that despite losing weight her daughter was spending long time
every day undertaking heavy exercises. Which of the following supports a diagnosis of
anorexia nervosa in this patient:Options:
Choose 5
221
PEDIATRICS
EXAMINATION REVIEW
L. ECHO
M. Neurophysiology
Instructions: For each of the patients described below choose the single most likely option from the
above list of options. Each option may be used once, more than once or not at all.
1. A 5-year-old Asian refugee child develops high fevers, organomegaly, joint pains and a rash.
He also has a 4/6 murmur.
2. A 45-year-old man who was on a fast developed acute pain in his toe.
3. A 45-year-old woman presents with wrist pain, which is worse at night and relieved by
dangling the hand over the edge of the bed and shaking it.
4. A 45-year-old man presents with swollen, painful and stiff hands and feet and the symptoms
are worse in the mornings.
5. A man has a bike accident and injures his leg. There is a small open wound. The wound is
treated but 2 days later he develops pain in his knee.
Teaching Notes for Question 11
Theme: Differential diagnosis of joint pains
1. This is rheumatic fever. He should have a full cardiac assessment.
2. Joint aspiration and microscopy is needed here as this is a case of gout which is due to deposition of
sodium monourate crystals in the joints and may be precipitated by trauma, surgery, starvation,
infection and diuretics. Diagnosis depends upon finding urate crystals in tissues and synovial fluid.
3. Neurophysiology is needed here as this is a case of carpal tunnel syndrome and neurophysiology to
find the level of the lesion and assess axonal degeneration and the likelihood of improvement after
surgery is essential.
4. Rheumatoid factor and joint X-ray is needed here as this is a case of rheumatoid arthritis which
presents with swollen, painful and stiff hands and feet and the symptoms are worse in the mornings.
5. Joint aspiration, swab and blood culture is needed here as this is a case of infection of the joint
which may have gained entry from the wound.
12-Options
A. Varicella
B. Scabies
C. Pityriasis rosea
D. Psoriasis
E. Meningitis
F. Measles
G. Kawasakis disease
H. Herpes simplex virus
I. Ebstein-Barr virus
J. Roseola infantum
K. Impetigo
L. Erythema toxicum
Instructions: For each of the patients described below choose the single most likely option from the
above list of options: Each option may be used once, more than once or not at all.
1. A 6-month-old boy presents with a generalised rash after a bout of fever.
2. A 3-year-old boy presents with oedema, congested conjunctivae, erythema of the oral
cavity, lips, and palms. It does not respond to antibiotics.
3. A mother rings A&E saying that her 3-year-old boy has fever for 1 day and is drowsy. He
also has a rash which does not blanch on pressure.
4. A 17-year-old African male presents with sore throat, fever, anorexia, malaise,
lymphadenopathy, palatal petechiae, and splenomegaly.
5. A non-immunised child of 5 years presents with catarrh, conjunctivitis, fever. He has some
bluish white spots on his buccal mucosa.
222
PEDIATRICS
EXAMINATION REVIEW
223
PEDIATRICS
EXAMINATION REVIEW
D. Brucellosis
E. Tuberculosis
F. Poliomyelitis
G. Candidiasis
H. Infectious mononucleosis
I. Rubella
J. Diphtheria
K. Coryza
L. Meningitis
M. Hepatitis
N. Influenza
Instructions: For each of the patients described below choose the single most likely option from the
above list of options: (Each option may be used once, more than once or not at all.)
1. A 4-year-old boy presents to A&E with shock. He is found to have myocarditis. His mother
gives a history of tonsillitis.
2. A 15-year-old patient presents with a rash and suboccipital lymphadenopathy. He was not
immunised as a child.
3. A 35-year-old lady presents with myalgia, nausea, and fever. She also has abdominal pain.
Her bilirubin is raised.
4. A veterinarian presents with fever, malaise, anorexia, vomiting, hepatosplenomegaly,
depression, arthritis, sacro-ileitis and rash.
5. A man who is on steroids for rheumatoid arthritis for a long period of time now presents
with white patches in his mouth.
Teaching Notes for Question 14
Theme: Diagnosis of infections
Q1: This is a case of diphtheria which is caused by corynebacterium diphtheriae. Diphtheriae usually
presents with a brassy cough, tonsillitis and a false membrane over the fauces. The toxin may cause
polyneuritis. Shock may occur from myocarditis, toxaemia or conducting system involvement.
Q2: This is a case of Rubella caused by a RNA virus. The incubation period 14-21 days. It presents with
a mild macular rash and suboccipital lymphadenopathy. Small joint arthritis may also be present.
Q3: This is a case of hepatitis. This is evident from the fact that the patient has jaundice (yellow
sclera) and pain in the region of his right upper abdomen.
Q4: This is a case of brucellosis which is common in the Middle East. Brucellosis is more commonly
found amongst vets and farmers. The following signs and symptoms are present: fever, malaise,
anorexia, vomiting, hepatosplenomegaly, depression, arthritis, sacro-ileitis and rash. A diagnosis is
made using blood cultures.
Q5: This is a case of candidiasis which is commonly found in patients who have low immunity.
224